FLT - 05 - Explanation Final

You might also like

Download as pdf or txt
Download as pdf or txt
You are on page 1of 58

VAJIRAM & RAVI

Institute for IAS Examination

Prelims Test Series - 2024


Full Length Test – 05
(R5534)

Answer Key
&
Detailed Answer Explanation
Prelims Test Series – Full Length Test – 05 (R5534) - Answer Key
1. (a) 11. (a) 21. (d) 31. (a) 41. (d) 51. (a) 61. (c) 71. (b) 81. (a) 91. (d)
2. (c) 12. (a) 22. (b) 32. (a) 42. (b) 52. (d) 62. (d) 72. (c) 82. (c) 92. (b)
3. (b) 13. (b) 23. (a) 33. (c) 43. (c) 53. (a) 63. (a) 73. (c) 83. (d) 93. (b)
4. (b) 14. (d) 24. (a) 34. (a) 44. (c) 54. (c) 64. (a) 74. (d) 84. (c) 94. (a)
5. (a) 15. (c) 25. (b) 35. (d) 45. (b) 55. (c) 65. (c) 75. (a) 85. (d) 95. (b)
6. (b) 16. (c) 26. (c) 36. (a) 46. (a) 56. (b) 66. (b) 76. (b) 86. (d) 96. (b)
7. (c) 17. (b) 27. (a) 37. (a) 47. (b) 57. (c) 67. (d) 77. (c) 87. (c) 97. (d)
8. (d) 18. (b) 28. (b) 38. (b) 48. (c) 58. (c) 68. (c) 78. (b) 88. (c) 98. (a)
9. (b) 19. (b) 29. (b) 39. (d) 49. (c) 59. (a) 69. (d) 79. (c) 89. (d) 99. (d)
10. (c) 20. (a) 30. (d) 40. (b) 50. (d) 60. (c) 70. (b) 80. (d) 90. (c) 100. (d)

Q1.
Answer: a
Explanation:
● Ordinarily, all business of the legislature requires that a motion or resolution or bill should get the
support of a majority of the members voting at that time.
Effective Majority:
● It refers to a numerical representation exceeding 50% of the effective strength of the House. The
effective strength is calculated by subtracting the number of vacancies from the total strength.
Situations where it is needed are as follows:
○ For the removal of the Vice President in the Rajya Sabha. So, point 1 is correct.
○ Resolution seeking the removal of Speaker and Deputy Speaker of Lok Sabha and State Legislative
Assemblies
○ Resolution seeking the removal of the Deputy Chairman of Rajya Sabha
Difference between Absolute and Effective Majority:
● Absolute majority means more than 50% of the total strength of the House irrespective of the vacancies
whereas, an effective majority is the majority of the total strength of the house after subtracting
vacancies from it.
Special Majority:
● According to Article 249 (when Rajya Sabha legislates in matters of State list) and Article 312 of the
Constitution of India (when Rajya Sabha creates new All India Services), a special majority means 2/3rd
of the members present and voting. So, point 2 is not correct.
● According to Article 368, a special majority means 2/3rd of the members present and voting plus the
majority of the total membership of the house (Absolute majority). Special Majority under Article 368 is
needed for the following:
○ Amendment of the Constitution
○ Removal of a judge of the Supreme Court or a High Court. So, point 3 is not correct.
○ For the approval of the National Emergency. So, point 4 is not correct.
1
Vajiram & Ravi Prelims Test Series (2024)
Full Length Test – 05 – (R5534)
○ To create or abolish the Legislative Council in a State
So, only one of the above requires an ‘effective majority’ of each House of the Parliament.
Therefore, option (a) is the correct answer.

Q2.
Answer: c
Explanation:
● The authority of the Constitution of India is higher than that of the Parliament. The Parliamentary
System in India is modelled on the ‘Westminster model of government’. However, unlike in Britain,
Parliament in India is not sovereign under the Constitution of India. The Constitution of India is
sovereign and the Parliament is the creature of the Constitution and derives all its powers from the
Constitution. So, statement 1 is correct.
● The Constitution acts as a boundary wall for the Parliament as it cannot use its power beyond the level
given to it by the Constitution. Although the Parliament can amend the Constitution, it cannot alter the
Basic Structure of the Constitution. So, statement 2 is not correct.
● The Constitution of India specifies how Parliament is to be formed and what are its powers. Articles 79
to 122 of the Constitution of India deals with the organisation, composition, duration, officers,
procedures, privileges, powers and so on of the Parliament. Thus, it is the source of authority for the
Parliament.
So, statement – I is correct but Statement – II is incorrect.
Therefore, option (c) is the correct answer.

Q3.
Answer: b
Explanation:
● Article 14 of the Constitution of India declares that 'the State shall not deny to any person equality before
the law or the equal protection of the laws within the territory of India'.
● The guarantee of equality before the law is an aspect of what is called the rule of law in England. It means
that no man is above the law and that every person, whatever be his rank or conditions, is subject to the
jurisdiction of ordinary courts.
○ The above rule of equality is, however, not an absolute rule and there are a number of exceptions
to it.
○ The rule of law does not prevent certain classes of persons being subject to special rules. Thus,
members of the armed forces are controlled by military laws. Similarly, medical practitioners are
subjected to the regulations framed by the Medical Council of India, a statutory body. So,
statement 1 is correct.
○ Equality before the law does not mean that the "powers of the private citizens are the same as
the powers of the public officials". Thus, a police officer has the power to arrest while, as a
general rule, no private person has this power. So, statement 2 is not correct.
○ Thirdly, Ministers and other executive bodies are given very wide discretionary powers by the
statutes. A Minister may be allowed by law to act as he thinks fit' or 'if he is satisfied. So,
statement 3 is correct.
2
Vajiram & Ravi Prelims Test Series (2024)
Full Length Test – 05 – (R5534)
○ Fourthly, certain members of society are governed by special rules in their professions, i.e.,
lawyers, doctors, nurses, members of armed forces and police. Such classes of people are treated
differently from ordinary citizens.
So, only two of the above statements are correct.
Therefore, option (b) is the correct answer.

Q4.
Answer: b
Explanation:
State Legislative Assembly:
● It consists of representatives directly elected by the people on the basis of universal adult franchise.
However, some members of the Legislative Assemblies in Sikkim and Nagaland are also elected
indirectly.
● Its maximum strength is fixed at 500 and minimum strength at 60. It means that its strength varies from
60 to 500 depending on the population size of the State. However, in case of Arunachal Pradesh, Sikkim
and Goa, the minimum number is fixed at 30 and in case of Mizoram and Nagaland, it is 40 and 46
respectively. So, statement 1 is correct and statement 2 is not correct.
● According to the Representation of People Act (1951), a person to be elected to the Legislative
Assembly must be an elector for an assembly constituency in the concerned State. So, statement 3 is
correct.
So, only two of the above statements are correct.
Therefore, option (b) is the correct answer.
Relevance: Elections to the Legislative Assemblies in five States recently concluded in India.

Q5.
Answer: a
Explanation:
● The Arbitration Council of India (ACI) is an independent statutory body created under the Arbitration
and Conciliation (Amendment) Act 2019. So, statement 1 is correct.
● It is established for the purpose of grading of arbitral institutions and accreditation of arbitrators, etc.
and not to encourage settlement of international disputes of India by arbitration as stated in the
Directive Principles of State Policy given under Article 51 of the Constitution of India. So, statement 2 is
not correct.
Membership of ACI:
● It is headed by a Chairperson, who has been a Judge of the Supreme Court or a Chief Justice or Judge of
a High Court or an eminent person, having special knowledge and experience in the conduct or
administration of arbitration, to be appointed by the Central Government in consultation with the Chief
Justice of India.
● Besides, it will also have two Full-time Members from amongst eminent arbitration practitioners and
academicians. In addition, one representative of a recognized body of commerce and industry shall be
nominated on rotational basis as a Part-time Member.

3
Vajiram & Ravi Prelims Test Series (2024)
Full Length Test – 05 – (R5534)
● The Secretary, Department of Legal Affairs, Ministry of Law & Justice; Secretary, Department of
Expenditure, Ministry of Finance and Chief Executive Officer, ACI will be ex-officio Members.
Therefore, option (a) is the correct answer.
Knowledge Box

● India’s foreign policy aims at the promotion of international peace and security.
● Article 51 of the Constitution (Directive Principles of State Policy) directs the Indian
State to:
○ promote international peace and security,
○ maintain just and honorable relations between nations,
○ foster respect for international law and treaty obligations, and
○ encourage settlement of international disputes by arbitration.
Relevance: The Indian Council of Arbitration (ICA) and the Vienna International Arbitral Centre (VIAC) entered
into a Memorandum of Understanding (MOU).

Q6.
Answer: b
Explanation:
The Principles of Natural Justice:
● They are a set of ‘procedural constraints’ that apply primarily to administrative decision-making in
India. Administrative action in India can be challenged on grounds, such as unreasonableness and,
increasingly on grounds of proportionality, arbitrariness, and, finally, procedural impropriety, i.e. for
violations of the principles of natural justice.
● By linking constitutional rights to the principles of natural justice, the Supreme Court has been able to
use these principles to review not only administrative action but also legislative enactments on these
grounds: either by reading these principles as constituent parts of several Fundamental Rights or using
them as interpretative tools in understanding these rights. So, statement 1 is correct.
○ The Supreme Court of India has been largely unanimous in agreeing that natural justice
encompasses two key principles: the Right to a Fair Hearing (audi alteram partem) and the rule
against bias (nemo iudex in sua causa). So, statement 3 is correct.
● A claim that Fundamental Rights in India included within their scope the principles of natural justice was
categorically rejected in one of the Supreme Court’s earliest decisions in 1950: A.K. Gopalan v. State
of Madras. So, statement 2 is not correct.
○ The case of Maneka Gandhi v. Union of India (1978) deals with the basic principles of natural
justice enshrined in the Constitution of India in the form of Fundamental Rights under Articles
14, 19 & 21.
So, only two of the above statements are correct.
Therefore, option (b) is the correct answer.
Knowledge Box

4
Vajiram & Ravi Prelims Test Series (2024)
Full Length Test – 05 – (R5534)
● As administrative law principles, natural justice operates as common law procedural
constraints on decision-making processes, mandating, for instance, the right to be
heard, to a reasoned decision, to cross-examine and to have a hearing that is free
from bias.
● Administrative actions are often reviewed by the judiciary because the principles of
natural justice have not been complied with; for instance, a common judicial
formulation on the jurisdiction of Indian High Courts specifically mentions “violation of
the principles of natural justice” as a ground for issuing the writ of certiorari.
○ The writ of certiorari is issued by a higher court to a lower court or tribunal
either to transfer a case pending with the latter to itself or to squash the order
of the latter in a case. It is issued on the grounds of excess of jurisdiction or lack
of jurisdiction or error of law.
○ The review of administrative action on the grounds of fairness, arbitrariness,
reasonableness, and proportionality is sometimes invoked under the
Fundamental Right to Equality, under Article 14 of the Constitution of India,
and to reasonable controls on several freedoms under Article 19 of the
Constitution.
○ In addition to this, the review of administrative action on substantive and
procedural grounds (in other jurisdictions, a “due process” requirement) is
linked to Article 21, which guarantees the right to life and personal liberty
subject to “procedure established by law.”
Relevance: The Delhi High Court recently observed that the principles of natural justice are not “mantras” but
foundational precepts concerned with fairness of procedure and the right of a person to respond to the
allegations made.

Q7.
Answer: c
Explanation:
President's Medal for Distinguished Service:
● It is given to recognize a special distinguished record or prolonged service, but only when distinguished
by very exceptional ability and merit or exhibition of conspicuous devotion to duty. So, statement 1 is
correct.
● Eligibility: Prolonged service of 25 years irrespective of rank on the date of occasion i.e. 26th January/
15th August as the case may be, marked by exceptional ability and merit. So, statement 2 is correct.
● The Central Award Committee headed by the Union Home Secretary considers and recommends the
nominations. Approving Authority of the award is the President of India. So, statement 3 is correct.
So, all three of the above statements are correct.
Therefore, option (c) is the correct answer.

5
Vajiram & Ravi Prelims Test Series (2024)
Full Length Test – 05 – (R5534)
Q8.
Answer: d
Explanation:
● The State Emblem of India (Prohibition of Improper Use) Act, 2005 primarily aims to prevent the
improper use of the State Emblem of India for professional and commercial purposes. Unauthorized
use of the State Emblem for any purpose other than national or official purposes is explicitly prohibited
under the Act to preserve its sanctity and prevent misuse.
● The Act has extraterritorial jurisdiction, meaning it applies to Indian citizens and entities both within
and outside the territorial boundaries of India. So, statement 1 is not correct.
● It is only the Central Government which is empowered by the Act to make rules for preventing the
improper use of the State Emblem, thereby providing necessary regulatory measures. So, statement 2
is not correct.
Therefore, option (d) is the correct answer.
Knowledge Box

● The State Emblem of India is an adaptation from


the Sarnath Lion Capital of Ashoka. The Lion
Capital has four lions mounted back to back on a
circular abacus.
● The profile of the Lion Capital showing three lions
mounted on the abacus with a Dharma Chakra in
the center, a bull on the right and a galloping
horse on the left, and outlines of Dharma Chakras
on the extreme right and left has been adopted as
the State Emblem of India. The bell-shaped lotus
has been omitted.
● The motto "Satyameva Jayate" – Truth alone
triumphs – written in Devanagari script below
the profile of the Lion Capital is part of the State
Emblem of India.

Relevance: The rear wheels of the Pragyan rover, which is sent on the Moon by the Indian Space Research
Organisation (ISRO), have been adorned with the ISRO logo and the national emblem, showcasing the Lion
Capital of Ashoka at Sarnath.

Q9.
Answer: b
Explanation:
Stare Decisis:
● It is the legal principle that courts should, where possible, adhere to precedent and not disturb settled
matters.
6
Vajiram & Ravi Prelims Test Series (2024)
Full Length Test – 05 – (R5534)
● This means that once a court has decided a legal issue, future cases involving the same issue should be
decided consistently. As a result, stare decisis promotes predictability in the law and enhances stability
and certainty for parties involved in legal disputes.
● However, courts may depart from precedent if the prior decision is clearly wrong or if circumstances
have changed significantly since the prior decision was made.
Therefore, option (b) is the correct answer.

Q10.
Answer: c
Explanation:
As per the Constitution of India, police and public order are State subjects under the Seventh Schedule.
However, the Union Ministry of Home Affairs (MHA) assists State Governments by providing them support of
the Central Armed Police Forces. The Ministry maintains seven Central Armed Police Forces (CAPF) given below:
● Assam Rifles (AR): They fulfill the dual role of maintaining internal security in the North-eastern region
and guarding the Indo-Myanmar Border.
○ They function under the administrative control of the Union Home Ministry while the
operational control lies with the Union Ministry of Defence. It is headed by Army personnel who
come under the Ministry of Defence. So, point 1 is correct.
● Sashastra Seema Bal (SSB): It mainly guards the Indo-Nepal and Indo - Bhutan Borders and is being
declared the Lead Intelligence Agency for these borders. So, point 2 is correct.
● Central Industrial Security Force (CISF): It protects vital installations (like airports, Delhi Metro Rail
Corporation, sensitive Government buildings and even heritage monuments) and public sector
undertakings. So, point 3 is correct.
● National Security Guard (NSG): It is a force especially equipped and trained to deal with specific
situations and is therefore to be used only in exceptional circumstances to thwart serious acts of
terrorism. So, point 5 is correct.
● Indian Coast Guard are not among the seven Central Armed Police Forces (CAPF). So, point 4 is not
correct.
○ It is a multi-mission organisation under the Ministry of Defence. Its mission is to protect India’s
ocean and offshore wealth including oil, fish and minerals. It also assists mariners in distress and
safeguard life and property at sea. It enforces maritime laws with respect to sea, poaching,
smuggling and narcotics and preserves marine environment and ecology and protects rare
species.
So, only four of the above security forces are part of the Central Armed Police Forces (CAPF).
Therefore, option (c) is the correct answer.
Knowledge Box

● Border Security Force (BSF): During peacetime, it promotes a sense of security among
the people living in the border areas and prevents trans-border crimes, unauthorised
entry into or exit from the territory of India.

7
Vajiram & Ravi Prelims Test Series (2024)
Full Length Test – 05 – (R5534)
○ During wartime, holding ground in less threatened sectors so long as the main
attack does not develop in a particular sector and it is felt that the local situation
is within the capability of BSF to deal with.
● Central Reserve Police Force (CRPF): It assists in internal security and
counterinsurgency operations.
● Indo Tibetan Border Police (ITBP): The entire stretch of India-China Border from
Karakoram Pass in Ladakh to Jachep La in Arunachal Pradesh comprising 3488 Kms is
assigned to the ITBP for border guarding duty and, accordingly, ITBP, replaced Assam
Rifles in Sikkim and Arunachal Pradesh in 2004.
Relevance: The Prime Minister of India recently paid tribute to the heroes of India's victory over Pakistan in the
1971 war on the occasion of 'Vijay Diwas' celebrated on 16 December every year.

Q11.
Answer: a
Explanation:
● E-visa allows travellers to apply for a
visa online and obtain it electronically
before travelling to a destination
country. This facilitates a more
convenient and streamlined visa
application process.
● E-visa can have different entry
specifications and some may be
limited to a single entry, while others
may allow multiple entries within a specified period. For example, e-visa is granted for a period up to
60 days with double entry on e-Tourist Visa and e-Business Visa and triple entry on e-Medical Visa. So,
statement 1 is not correct.
● E-Visa is granted to a foreigner whose sole objective of visiting India is recreation, sight-seeing, casual
visit to meet friends or relatives, attending a short term yoga programme, medical treatment including
treatment under Indian systems of medicine and business purpose and no other purpose/activity. So,
statement 2 is not correct.
○ This facility shall not be available if the person or either of his/her parents or grandparents
(paternal or maternal) was born in, or was permanently resident in Pakistan.
○ E-Visa facility shall not be available to holders of Diplomatic/Official passports, UNLP (UN
Passport) holders and international travel document holders e.g. INTERPOL officials. So,
statement 3 is correct.
● Persons holding e-Visa are allowed to enter into India only through the designated international
airports.
So, only one of the above statements is correct.
Therefore, option (a) is the correct answer.
Relevance: India has recently suspended all visa facilities for Canadian citizens.
8
Vajiram & Ravi Prelims Test Series (2024)
Full Length Test – 05 – (R5534)
Q12.
Answer: a
Explanation:
● A carbon budget is a cumulative amount of carbon dioxide emissions permitted over a period of time to
keep within a certain temperature threshold. It can also be defined as the amount of greenhouse gases
that can be ‘spent’ (emitted) for a given level of global warming.
● If this budget is exceeded, global temperatures will become higher. A global carbon budget determines
the input of carbon dioxide to the atmosphere by emissions from human activities, balanced by output
(storage) in the carbon reservoirs on land or in the ocean.
Therefore, option (a) is the correct answer.

Q13.
Answer: b
Explanation:
● The Maximum Residue Limit (MRL) is the maximum concentration of pesticide residue, given in mg/kg
of the commodity, that is legally permissible in or on food and animal feeds as a result of pesticide use
in accordance with Good Agricultural Practices (GAP).
● Under the joint Food and Agriculture Organisation/World Health Organisation (FAO/WHO) Food
Standards Programme, the MRLs were established at the international level by the Codex Alimentarius
Commission. So, statement 1 is correct.
● Each country sets its own MRL values; for example, in India, MRLs are determined by the Food Safety
and Standards Authority of India (FSSAI), Union Ministry of Health and Family Welfare, under the Food
Safety and Standards Authority Act of 2006. So, statement 2 is correct.
● MRLs have been established for a variety of raw and processed food commodities. Foods can be of
plant or animal origin, and they can be consumed by humans or animals. So, statement 3 is not correct.
So, only two of the above statements are correct.
Therefore, option (b) is the correct answer.

Q14.
Answer: d
Explanation:
● Numerous animal species naturally produce chemical toxins (called venom) which are used to kill or
incapacitate prey or as a defence against predators.
● Of the terrestrial venomous snakes, four species are widespread on the Indian mainland, also known as
the “big four.” They include spectacled cobra (Naja naja), common krait (Bungarus caeruleus), Russell’s
viper (Daboia russelii), and saw-scaled viper (Echis carinatus). These paramount species are responsible
for the majority of the mortality and injury cases in the country. So, points 1 and 2 are correct.
● Though the venom of most jellyfish is not harmful, some can be deadly. For example, the Indo-Pacific
box jellyfish, or sea wasp, releases venom that makes the heart contract. So, point 3 is correct.
● According to a new study, all octopuses, cuttlefish, and some squid are venomous. Octopuses are
venomous animals that have existed for 300 million years. So, point 4 is correct.
So, all four of the above animal species are venomous in nature.

9
Vajiram & Ravi Prelims Test Series (2024)
Full Length Test – 05 – (R5534)
Therefore, option (d) is the correct answer.

Q15.
Answer: c
Explanation:
● "Biofine" and "BioCRACK" technologies represent significant advancements in the realm of biofuel
production, focusing on converting biomass into renewable energy sources.
● Biofine Technology utilises a thermochemical process to convert lignocellulosic biomass, such as wood
and agricultural residues, into levulinic acid, a versatile platform chemical. This acid can further be
transformed into various biofuels and chemicals.
● BioCRACK Technology, on the other hand, is a novel approach that subjects biomass to a pyrolysis
process in the presence of vacuum gas oil. This results in the production of bio-oil, which can be refined
into different types of biofuels.
Therefore, option (c) is the correct answer.

Q16.
Answer: c
Explanation:
● The United Nations Conference on Environment and Development (UNCED), also known as Earth
Summit, was convened in Rio de Janeiro in June 1992 to promote economic development, reduce
poverty, and preserve and protect the earth’s ecological systems.
○ It led to the creation of new institutions for sustainable development, including the United
Nations Commission on Sustainable Development. So, point 1 is correct.
○ It led to the signing of two new environmental treaties, the United Nations Framework
Convention on Climate Change and the Convention on Biological Diversity. So, point 2 is correct.
○ It led to the adoption of the Rio Declaration on Environment and Development, Agenda 21, and
the Statement of Forest Principles. So, point 3 is correct.
● The Bonn Convention, also known as the Convention on the Conservation of Migratory Species of Wild
Animals (CMS), is a United Nations environmental treaty. It was signed in Bonn, Germany in 1979 and
aims to protect migratory species and their habitats. It emerged much earlier than the Earth Summit.
So, point 4 is not correct.
So, only three of the above were established as a result of the Earth Summit, 1992.
Therefore, option (c) is the correct answer.

Q17.
Answer: b
Explanation:
● Coal gasification is a process that converts coal and water into syngas. Syngas is a mixture of carbon
monoxide, hydrogen, carbon dioxide, methane, and water vapour.
● Coal gasification is a cleaner, less-polluting method of processing coal. It emits less carbon upon burning,
and other polluting gases like CO2 can be easily separated, captured, and put to other uses. So,
statement 1 is not correct.

10
Vajiram & Ravi Prelims Test Series (2024)
Full Length Test – 05 – (R5534)
● Coal gasification enables the production of syngas (a mixture of hydrogen and carbon monoxide) from
coal. Hydrogen extracted from this process can be separated and used as a clean fuel for various
applications, including power generation and as fuel for vehicles. So, statement 2 is correct.
● Gasification of coal is a process in which coal is partially oxidised by air, oxygen, steam or carbon dioxide
under controlled conditions to produce a fuel gas. Coal gasification processes can produce three types
of ash: fly ash (including char or unreacted fuel), bottom ash, and slag, with most of the solid byproduct
ending in the form of slag for high-temperature gasifiers. Non-slagging gasification produces a coarse
bottom ash and fine fly ash. So, statement 3 is correct.
So, only two of the above statements are correct.
Therefore, option (b) is the correct answer.
Relevance: According to the interim budget 2024, India aims to establish coal gasification and liquefaction
capacity of 100 million tonnes by 2030.

Q18.
Answer: b
Explanation:
● The Dandeli forest in Uttara Kannada district of Karnataka is known for its forest grasses, birds and cattle
herders. This forest is also home to a large number of wildlife like hornbills. The hornbill birds that this
area is famous for also feed on figs and they have been abandoning their nests in the past 4-5 years.
● There are around 100 different types of grasses that grow in the undergrowth of the Dandeli forest,
according to a survey published by scientists from the Centre for Ecological Sciences of the Indian
Institute of Science, Bengaluru. These are the primary sources of food for the herbivore population of
the forest such as barking deer, chital (spotted deer) and elephants. The grasses are being replaced by
eupatorium weed, which is not eaten by the herbivores and is prone to fires.
○ The grasses in the Dandeli area proliferated mainly during the colonial era when the lush semi-
evergreen and evergreen forests were replaced by monoculture teak plantations and the general
character of the forest also changed from semi-evergreen to moist deciduous.
Therefore, option (b) is the correct answer.
Relevance: The Dandeli forest is losing its characteristic grasses and hornbills to erratic weather.

Q19.
Answer: b
Explanation:
● Saprotrophs are organisms that feed on nonliving organic matter known as detritus at a microscopic
level. They are considered critical to decomposition and nutrient cycling and include fungi, certain
bacteria, and fungus-like organisms known as water molds (phylum Oomycota). So, points 1 and 2 are
correct.
○ They feed by a process known as absorptive nutrition, in which the nutritional substrate is
directly digested by a variety of enzymes that are excreted by the saprotroph. The enzymes
convert the detritus into simpler molecules, which are then absorbed by the cells to feed the
organism.

11
Vajiram & Ravi Prelims Test Series (2024)
Full Length Test – 05 – (R5534)
● Viruses are organisms that replicate inside a host's body. They are obligate parasites, meaning they lack
the metabolic machinery to generate energy or synthesize proteins. Instead, they depend on host cells
to carry out these vital functions. They are not saprotrophs. So, point 3 is not correct.
So, only two of the above organisms are examples of saprotrophs.
Therefore, option (b) is the correct answer.

Q20.
Answer: a
Explanation:
● Microalgae or microphytes are microscopic algae that are invisible to the naked eye. These are
phytoplankton commonly found in freshwater and marine systems, living in both the water column and
sediments. So, statement 1 is correct.
○ Photosynthesis in the ocean is dominated by microalgae, which together with cyanobacteria are
collectively known as phytoplankton.
● They are unicellular species that exist individually or in chains or groups. Unlike higher plants, microalgae
do not have roots, stems or leaves. They are particularly adapted to environments where viscous forces
dominate. So, statement 2 is not correct.
○ Microalgae are considered as potential feedstocks for biofuels and have also emerged as
promising microorganisms in bioremediation.
● All microalgae can grow photo-autotrophically, but few species can grow heterotrophically.
Heterotrophic microalgae cannot consume carbon dioxide, even though they generate CO 2 through the
metabolism of organic carbon. An exception among microalgae is the colorless Prototheca, which has
no chlorophyll at all. These leafless algae turn parasitic and cause a disease called protecosis in humans
and animals. So, statement 3 is not correct.
So, only one of the above statements is correct.
Therefore, option (a) is the correct answer.
Relevance: According to a new study, microalgae appear to rely on a unique strategy to cope with global
warming.

Q21.
Answer: d
Explanation:
● Lightning is a giant discharge of electricity accompanied by a brilliant flash of light and a loud crack of
thunder.
● It is not confined to thunderstorms. It's been seen in volcanic eruptions, extremely intense forest fires,
surface nuclear detonations, heavy snowstorms and in large hurricanes.
Lightning and Monsoon:
● Lightening is mainly associated with the convective clouds. So, Statement 2 is correct.
● Monsoon clouds are mainly stratiform. Therefore, lightning generally does not occur during the active
phase of the monsoon season. So, Statement 1 is not correct.
● However, the convective activity during the break spell of the monsoon may lead to formation of
convective clouds and hence the lightning.

12
Vajiram & Ravi Prelims Test Series (2024)
Full Length Test – 05 – (R5534)
So, Statement-I is incorrect but Statement-II is correct.
Therefore, option (d) is the correct answer.
Knowledge Box

Formation of lightning:
● Lightning is the result of the
difference in electrical charge
between the top and bottom of a
cloud.
● The lightning-generating clouds are
typically about 10-12 km in height,
with their base about 1-2 km from
the Earth’s surface. The
temperatures at the top range from
-35°C to -45°C.
● As water vapour moves upwards in
the cloud, it condenses into water due to decreasing temperatures. A huge amount of
heat is generated in the process, pushing the water molecules further up.
● As they move to temperatures below zero, droplets change into small ice crystals. As
they continue upwards, they gather mass, until they become so heavy that they start
descending.
● It leads to a system where smaller ice crystals move upwards while larger ones come
down. The resulting collisions trigger release of electrons, in a process very similar to
the generation of electric sparks. The moving free electrons cause more collisions and
more electrons; a chain reaction is formed.
● The process results in a situation in which the top layer of the cloud gets positively
charged while the middle/lower layer is negatively charged. Enormous charge
differences develop. In little time, a huge current starts to flow between the layers.
● It produces heat, leading to the heating of the air column between the two layers of
cloud. It is because of this heat that the air column looks red during lightning.
● The heated air column expands and produces shock waves that result in thunder
sounds.
Relevance: At least 24 people have been killed by lightning and intense rain and hail storms in western India.

Q22.
Answer: b
Explanation:
● The Sea of Galilee lies between Israel occupied Golan Heights and the Galilee region (Syria). Its basin lies
in Israel, Jordan and Syria. So pair 1 is not correctly matched.

13
Vajiram & Ravi Prelims Test Series (2024)
Full Length Test – 05 – (R5534)
● The Jordan river rises on the slopes of Mount Hermon and lies
between Syria and Lebanon and flows southward through
northern Israel to the Sea of Galilee. So, pair 2 is correctly
matched.
● Dead Sea is the landlocked salt lake between Israel and Jordan
in south-western Asia. Its eastern shore belongs to Jordan, and
the southern half of its western shore belongs to Israel. It has the
lowest elevation and is the lowest body of water on the surface
of Earth. So, pair 3 is correctly matched.
● Sea of Azov, inland sea situated off the southern shores of
Ukraine and Russia. So, pair 4 is not correctly matched.
So, only two of the above pairs are correctly matched.
Therefore, option (b) is the correct answer.
Relevance: Due to the recent conflict between Israel - Palestine and
Russia – Ukraine water bodies in these regions are often seen in the
news.

Q23.
Answer: a
Explanation:
Chilika Lake:
● It is located in Odisha and is a shallow lagoon with estuarine character. It is separated from the Bay of
Bengal by a narrow spit.
● It is the largest brackish water lagoon in India.
● It lies between the deltas of Mahanadi and Godavari rivers. So, point 1 is not correct.
● It is the largest wintering ground for migratory waterfowl found anywhere on the Indian sub-continent.
The endangered Irrawaddy dolphins are also found in the lake, which is the single largest habitat of this
species in the world.
Kolleru Lake:
● It is located in northeastern Andhra Pradesh. It lies
between the Godavari and Krishna River deltas. So,
point 2 is correct.
● It is one of the largest freshwater lakes in India. It was
designated a Wildlife Sanctuary in 1999.
Pulicat Lake:
● It is the second largest brackish water lake or lagoon
in India, after Chilika Lake.
● It is located on the Coromandel Coast of Andhra
Pradesh and lies between the deltas of Penner and
Palar rivers. So, point 3 is not correct.

14
Vajiram & Ravi Prelims Test Series (2024)
Full Length Test – 05 – (R5534)
● The long and narrow Sriharikota Island, which separates Pulicat Lake from the Bay of Bengal, is the site
of Satish Dhawan Space Centre, India’s satellite-launching facility.
So, only one of the above lakes lie between the deltas of Godavari and Krishna rivers.
Therefore, option (a) is the correct answer.
Knowledge Box

Lagoons Vs Estuaries:
● Lagoons are a type of wetland that occur in coastal enclosures that are shallow and
separated from the sea by a sand bar or strip of land. Occasionally they may receive sea
water, but do not experience typical tidal fluctuations of an estuary.
● An estuary is a partially enclosed coastal body of brackish water with one or more rivers
or streams flowing into it and with a free connection to the open sea. Estuaries form a
transition zone between river environments and maritime environments and are an
example of an ecotone.
International Network of Living Lakes:
● Since 2004, Pulicat Lake has been a member of the International Network of Living
Lakes along with Wular Lake and Chilika Lake.
● The Living Lakes is an international network program managed by the Global Nature
Fund (GNF) to enhance the protection, restoration and rehabilitation of lakes,
wetlands, other freshwater bodies of the world and their catchment areas.
Relevance: The survival of Kolleru Lake is under severe threat as aqua ponds continue to encroach and thrive in
the heart of the wetland.

Q24.
Answer: a
Explanation:
● Tropical cyclone is an intense circular storm that originates
over warm tropical oceans and is characterised by low
atmospheric pressure, high winds and heavy rain.
● Due to the Coriolis force, tropical cyclones rotate in a
counterclockwise direction in the Northern Hemisphere and
in a clockwise direction in the Southern Hemisphere.
○ Coriolis force is an apparent force caused by the
rotation of the Earth.
○ Coriolis force deflects the wind to the right direction
in the Northern Hemisphere and to the left in the
Southern Hemisphere.
● Tropical cyclones rarely if ever form between 5 degree North and 5 degree South latitudes,
respectively.
● Even under ideal conditions, a tropical cyclone is not likely to form if it is not at least 300 or so miles
from the equator. This is because of the lack of the Coriolis force at the equator.

15
Vajiram & Ravi Prelims Test Series (2024)
Full Length Test – 05 – (R5534)
● A hurricane cannot cross the equator if a cyclone has already formed beyond 5 degrees latitude:
○ Though theoretically possible, because a well-developed
storm has plenty of spin that would dominate the weak
Coriolis force near the equator.
○ However, practically it is not likely as if it crosses the
equator, the Coriolis force will start working against the
initial direction of the spin. So, statements 1 and 2 are
correct.
So, both Statement-I and Statement-II are correct and Statement-II is
the correct explanation for Statement-I.
Therefore, option (a) is the correct answer.
Knowledge Box

Different names of Tropical Cyclones:


● Tropical cyclones are known by various names in different parts of the world. In the
North Atlantic Ocean and the eastern North Pacific they are called hurricanes and in
the western North Pacific around the Philippines, Japan, and China the storms are
referred to as typhoons.
● In the western South Pacific and Indian Ocean they are variously referred to as severe
tropical cyclones, tropical cyclones, or simply cyclones.
Prerequisite conditions for cyclone formation:
● A warm, deep ocean waters (greater than 80°F / 27°C), an atmosphere cooling rapidly
with altitude, moist middle layers of the atmosphere, low wind shear, and a pre-existing
near surface disturbance.
Relevance: Cyclone Michaung made landfall over Nellore in Andhra Pradesh as a super-cyclonic storm.

Q25.
Answer: b
Explanation:
● Waste/Biomass sources like agricultural residue, cattle dung, sugarcane press mud, municipal solid
waste and sewage treatment plant waste, etc. produce biogas through the process of anaerobic
decomposition.
● The biogas is then purified to remove (and not as an essential component) hydrogen sulfide, carbon
dioxide, water vapor and compressed as Compressed BioGas (CBG), which has methane content of
more than 90%. So, statement 1 is not correct
● It has calorific value and other properties similar to Compressed Natural Gas (CNG) and hence can be
utilized as green renewable automotive fuel. Thus it can replace CNG in automotive, industrial and
commercial areas, given the abundance of biomass availability within the country. So, statement 2 is
correct.
● It can be transported through cylinder cascades or pipelines to retail outlets. So, statement 3 is correct.
So, only two of the above statements are correct.
Therefore, option (b) is the correct answer.
16
Vajiram & Ravi Prelims Test Series (2024)
Full Length Test – 05 – (R5534)
Knowledge Box

Conversion of agricultural residue, cattle dung and municipal solid waste (MSW) into CBG in a
commercial scale is expected to have the following benefits:
○ Contribution towards Swachh Bharat Mission through responsible waste
management.
○ Import reduction of natural gas and crude.
○ Utilization of agricultural residue, cattle dung and MSW for the production of
CBG and thus to achieve reduction in emissions and pollution.
○ A boost towards fulfillment of National commitments in achieving climate
change goals.
○ Providing a buffer against energy security concerns and crude/gas price
fluctuations.
○ Lowering pollution and carbon emission.
○ Providing additional source of revenue to the farmers, rural employment and
amelioration of the rural economy.

Relevance: Giants of the energy and environmental sectors showing increasing interest and investment in
Compressed BioGas (CBG).

Q26.
Answer: c
Explanation:
● Aurora borealis or Northern lights which are usually seen in polar regions are actually billions of charged
particles moving into space at ultra-high speeds. When these charged particles arrive in the direction of
Earth, they cause a disturbance in the Earth's magnetic field and enable us to see the aurora lights.
● A Stable Auroral Red (SAR) arc is not a conventional aurora. A SAR arc is a band of reddish light seen in
the sky. Unlike auroras where various colours appear in moving patterns, SAR displays are static and
monochromatic. So, statement 3 is correct.
● Both appear during periods of geomagnetic activity triggered by a wave of charged matter blasted out
of the Sun, but their mechanism of formation is slightly different. So, statement 1 is correct.
● Auroras appear when charged particles rain down from space, hitting the atmosphere and causing it to
glow. SAR arcs form differently. They are generated by extreme thermal and kinetic energy in Earth’s
atmosphere and are a sign of heat energy leaking into the upper atmosphere from Earth’s ring current
system. So, statement 2 is correct.
So, all three of the above statements are correct.
Therefore, option (c) is the correct answer.
Knowledge Box

Stable Auroral Red (SAR) arcs and Strong Thermal Emission Velocity Enhancement (STEVE)
phenomena:

17
Vajiram & Ravi Prelims Test Series (2024)
Full Length Test – 05 – (R5534)
● They are optical structures that have
been detected in the subauroral
upper atmosphere.
● The two optical structures differ
from each other in terms of spectra,
size and duration; a SAR arc can
evolve into a STEVE phenomenon
during a geomagnetic storm.
Fig: A SAR arc (left) appeared over New
Zealand in 2015 and half an hour later it evolved into a STEVE (right).
Relevance: A Stable Auroral Red (SAR) arc, a very rare event, was recently captured in Ladakh.

Q27.
Answer: a
Explanation:
Gresham’s law:
● It came into play most recently during the economic crisis in Sri Lanka last year, during which the Sri
Lankan Central Bank fixed the exchange rate between the Sri Lankan rupee and the U.S. dollar.
● Named after English financier Thomas Gresham, it suggests that bad money (devalued or less reliable
currency) tends to drive out good money (more stable and reliable currency).
● It comes into play when the exchange rate between two currencies is fixed by the Government at a
certain ratio that is different from the market exchange rate. Such price fixing causes the undervalued
currency, that is, the currency whose price is fixed at a level below the market rate, to go out of
circulation. The overvalued currency, on the other hand, remains in circulation but it does not find
enough buyers.
● Gresham’s law applies not just to paper currencies but also to commodity currencies and other goods.
○ In fact, whenever the price of any commodity, whether it is used as money or not, is fixed
arbitrarily such that it becomes undervalued when compared to the market exchange rate, this
causes the commodity to disappear from the formal market.
○ The only way to get hold of an undervalued commodity in such cases would be through the black
market.
Therefore, option (a) is the correct answer.
Knowledge Box

Thiers’ Law:
● Gresham’s law, however, holds true only when the exchange rate between currencies
is fixed under law by the Government and the law is implemented effectively by
authorities.

18
Vajiram & Ravi Prelims Test Series (2024)
Full Length Test – 05 – (R5534)
○ In the absence of any Government decree fixing the exchange rate between
currencies, it is good money that eventually drives bad money out of the market
and not the other way round.
● When the exchange rate between currencies is not fixed and people have the choice to
freely choose between currencies, people gradually stop using currencies that they
consider to be of poor quality and adopt currencies that are found to be of better
quality. This phenomenon wherein “good money drives out bad” is called Thiers’ Law
(named after French politician Adolphe Thiers) and it is seen as a complement to
Gresham’s law.
○ The rise of private cryptocurrencies in recent years has been cited by many
analysts as an example of good money issued by private money producers
driving out bad money issued by governments.
Relevance: Gresham’s law came into play most recently during the economic crisis in Sri Lanka last year, during
which the Sri Lankan central bank fixed the exchange rate between the Sri Lankan rupee and the U.S. dollar.

Q28.
Answer: b
Explanation:
Hard currency:
● It is a stable and reliable form of currency that is issued by the government and widely accepted around
the world. Soft currency is an unstable form of currency that is unconvertable, fluctuates erratically,
and/or depreciates against other currencies.
● It can be widely accepted around the world as a form of payment for goods and services because of its
convertibility. So, statement 1 is correct.
● Hard currencies mostly exist in developed countries with relatively stable governments with a stable
economy. whereas Soft currencies mostly exist in developing countries with relatively unstable
governments. So statement 2 is not correct.
● Hard currency is highly liquid in the foreign exchange (FX) market due to its convertibility, high volume
trade and large supply in the market. So statement 3 is not correct.
● In a country with Hard currency, the government generally does not manipulate the currency and allows
its rate as per market demand and supply. Therefore it is easily convertible to other currencies. So
statement 4 is correct.
So, only two of the above are correct if a currency is given the status of a ‘hard currency’.
Therefore, option (b) is the correct answer.
Relevance: According to a member of the Prime Minister’s Economic Advisory Council, by around 2030, Indian
rupee will be a global hard currency.

Q29.
Answer: b
Explanation:

19
Vajiram & Ravi Prelims Test Series (2024)
Full Length Test – 05 – (R5534)
● The Reserve Bank of India (Credit Card and Debit Card–Issuance and Conduct) Directions, 2022 provide
a thorough set of instructions primarily to card-issuers about issuing credit and debit cards, co-branded
cards, billing, and telemarketing, among other matters.
● The directions earmark which banks may engage in the credit and debit card business. Most Scheduled
Commercial Banks (SCBs) with a net worth of Rs. 100 crores can issue credit cards. The exception is
Regional Rural banks (RRBs) which need to collaborate with other banks to do so. So, statement 1 is not
correct.
● Urban Cooperative Banks (UCBs) with a net worth of more than Rs. 100 crores can issue credit cards
subject to certain guidelines. For example, they can only issue credit cards to members. They cannot
issue co-branded credit cards, and the total unsecured loans and advances given by a UCB cannot exceed
10 percent of its assets. So, statement 2 is correct.
● Non-Banking Financial Companies (NBFCs) registered with the Reserve Bank of India (RBI) with a
minimum net owned fund of Rs 100 crores can issue credit cards, provided they have a Certificate of
Registration and permission to enter the business. So, statement 3 is not correct.
○ Banks do not need RBI approval to issue co-branded debit or credit cards. UCBs cannot however
issue debit or credit cards in tie-ups with other non-bank entities.
● All banks can issue debit cards, without needing the approval of the RBI. However, debit cards can only
be issued to customers with Savings Bank/Current Accounts, and not to cash credit/loan account holders.
Banks may still link ‘the overdraft facility provided along with Pradhan Mantri Jan Dhan Yojana accounts
with a debit card’. So, statement 4 is correct.
So, only two of the above statements are correct.
Therefore, option (b) is the correct answer.
Relevance: According to the latest Reserve Bank of India data, credit card spending touched a record ₹1.78
trillion in October 2023 aided by strong festive season purchases on e-commerce platforms.

Q30.
Answer: d
Explanation:
● The Purple Economy, also sometimes referred to as the care economy, obtains its name from the colour
adopted by many feminist movements. It is a multidisciplinary approach to economics that encompasses
a diverse array of key social issues that improve everyone’s quality of life.
● This includes care activities and services, such as education, healthcare and women’s empowerment. It
champions the interests of vulnerable groups like children, the elderly and people with disabilities.
● It is an economic order organised around sustainability of caring labour through a redistributive
internalisation of the costs of care into the workings of the system.
Therefore, option (d) is the correct answer.
Knowledge Box

20
Vajiram & Ravi Prelims Test Series (2024)
Full Length Test – 05 – (R5534)
● The term white economy includes all sectors that involve the production, research,
marketing and distribution of health-related goods and services.
● The red economy model is focused on reducing production costs by employing a
linear business model of extracting resources and producing waste. The economy
is about mass production and mass consumption, it is derived from Fordism, which
was named after Henry Ford who operated as though environmental resources
were unlimited.
● Silver economy is the sum of the economic activities of people over the age of 50.
This includes the products and services they purchase and the economic activity
this spending generates.
Relevance: The organisation EnAble India, has been working on creating a ‘purple economy,’ which promotes
disability inclusion and job opportunities on a large scale.

Q31.
Answer: a
Explanation:
● Liquidity trap is a situation when expansionary monetary policy (increase in money supply) does not
increase the interest rate, income and hence does not stimulate economic growth.
● Low interest rates affect bondholder behaviour, especially when combined with concerns regarding the
current financial state of the nation. The end result is the selling of bonds at a level that is harmful to the
economy. Very low interest rates (at or close to 0%) thus lead to liquidity traps in an economy. So, point
1 is not correct.
● A persistent recessionaa causes deflation that raises real interest rate and lowers output even further
while monetary policy is ineffective, leading to liquidity trap. So, point 2 is correct.
● Ineffective expansionary monetary policy is also a characteristic of a liquidity trap. A liquidity trap is
usually caused by, and in turn perpetuates, deflation. When deflation is persistent and combined with
an extremely low nominal interest rate, it creates a vicious cycle of output stagnation and further
expectations of deflation that lead to a higher real interest rate. So, point 3 is not correct.
● A liquidity trap occurs when interest rates are very low but savings is high. In other words, consumers
and businesses are holding onto their cash even with the incentive of interest rates at or close to 0%. So,
point 4 is not correct.
So, only one of the above is the characteristics of a liquidity trap.
Therefore, option (a) is the correct answer.
Relevance: According to economists, China's economy might have been entered into a "liquidity trap".

Q32.
Answer: a
Explanation:
● The Direct Benefit Transfer (DBT) aims to transfer the benefits and subsidies of various social welfare
schemes directly in the bank account of the beneficiary on time by bringing efficiency, effectiveness,
transparency and also to eliminate the intermediary body.

21
Vajiram & Ravi Prelims Test Series (2024)
Full Length Test – 05 – (R5534)
● DBT transfers help expedite the flow of funds and information securely while reducing the possibility of
fraud. It eliminates the need for intermediaries, including government officers, in transferring the
subsidy amount directly into the beneficiary accounts.
● Beneficiaries can link only one bank by seeding the fund deposits to their Aadhaar details to avoid
duplication of subsidies. So, statement 1 is not correct.
○ The Electronic Payment Framework is to be followed by all Ministries/Departments and their
attached Institutions/PSUs and is applicable on all Central Sector (CS)/Centrally Sponsored
Schemes (CSS) and for all schemes where components of cash are transferred to individual
beneficiaries.
● Aadhaar is not mandatory in DBT schemes. Since Aadhaar provides a unique identity and is useful in
targeting the intended beneficiaries, it is preferred and beneficiaries are encouraged to have Aadhaar.
So, statement 2 is not correct.
● The Ministry of Finance (MoF) decided on mandatory use of the Public Financial Management System
(PFMS) of the office of Controller General of Accounts (CGA) for payment, accounting and reporting
under DBT. In 2014, MoF directed all implementing Ministries/Departments to ensure that no payments
under DBT schemes were processed unless the electronic payment files for such payments were received
through PFMS from 2015. Also, all DBT transactions have to be routed through the National Payment
Corporation of India (NPCI). A transaction cost of Rs. 0.50/- would be payable for each transaction to be
shared between the sponsor banks, destination entities and NPCI in the ratio of Rs. 0.10, Rs. 0.25 and
Rs. 0.15 respectively. So, statement 3 is correct.
So, only one of the above statements is correct.
Therefore, option (a) is the correct answer.
Relevance: According to recent data, the Central government has transferred subsidies and other payments to
the beneficiaries through the Direct Benefit Transfer (DBT) worth about Rs 3.2 trillion so far in the FY 2023-24.

Q33.
Answer: c
Explanation:
Securities Appellate Tribunal (SAT):
● It is formed as a statutory and autonomous body as per the provisions of the Securities and Exchange
Board of India (SEBI) Act, 1992. It was mainly established to hear appeals against orders passed by the
SEBI or by an adjudicating officer under the SEBI Act. So, point 1 is correct.
● The SAT hears appeals against the following orders:
○ Orders issued by the Pension Fund Regulatory and Development Authority (PFRDA) in relation
to cases filed before it. So, point 2 is correct.
○ Orders issued by the Insurance Regulatory and Development Authority of India (IRDAI) in
relation to cases filed before it. So, point 4 is correct.
● The National Company Law Appellate Tribunal (NCLAT) hears appeals against the orders passed by the
Insolvency and Bankruptcy Board of India. So, point 3 is not correct.
● SAT has the same powers as vested in a civil court under the code of civil procedure while trying a suit.
So, the Securities Appellate Tribunal (SAT) hears and disposes of appeals against orders passed by only three
of the above bodies.
22
Vajiram & Ravi Prelims Test Series (2024)
Full Length Test – 05 – (R5534)
Therefore, option (c) is the correct answer.
Knowledge Box

Who can make an appeal to the Securities Appellate Tribunal (SAT)?


● Every person aggrieved by the order of the Securities and Exchange Board of India (SEBI)
or adjudicating officer is liable to make an appeal to the SAT.
● No appeal can be made to the SAT against any order made with the consent of the
parties.
Appeal against the orders of the SAT:
● Every person aggrieved by any order or decision of SAT can file an appeal to the
Supreme Court. An appeal can only be made on a question of law.
Composition of the SAT:
● It consists of one Presiding Officer and such number of Judicial and Technical members
as the Central Government may determine.
● The person so appointed as the Presiding Officer should meet the following
requirements:
○ The retired or sitting judge of the Supreme Court
○ Chief Justice of the High Court
○ Judge of the High Court, who has completed at least seven years of service as a
judge in a high court.
Relevance: The Securities Appellate Tribunal has refused to interfere with the Securities and Exchange Board of
India’s interim order in connection with the Brightcom Group’s case.

Q34.
Answer: a
Explanation:
● Under the regressive taxation system of taxation, the tax rate diminishes as the taxable amount
increases. In other words, there is an inverse relationship between the tax rate and taxable income. The
rate of taxation decreases as the income of taxpayers increases. This system of taxation generally
benefits the higher sections of the society having higher incomes as they need to pay tax at lesser rates.
On the other hand, people with lesser incomes are burdened with a higher rate of taxation. Hence, all
indirect taxes are considered to be regressive in nature. So, statements 1 and 2 are correct.
So, both Statement–I and Statement–II are correct and statement–II is the correct explanation for Statement–
I.
Therefore, option (a) is the correct answer.
Knowledge Box

23
Vajiram & Ravi Prelims Test Series (2024)
Full Length Test – 05 – (R5534)
● Progressive tax is the taxing mechanism in which the taxing authority charges
more taxes as the income of the taxpayer increases. A higher tax is collected from
the taxpayers who earn more and lower taxes from taxpayers earning less. The
government uses a progressive tax mechanism.
● Proportional tax is the taxing mechanism in which the taxing authority charges the
same rate of tax from each taxpayer, irrespective of income. This means that lower
class, or middle class, or upper class people pay the same amount of tax. Since
the tax is charged at a flat rate for everyone, whether earning higher income or
lower income, it is also called flat tax.
Relevance: According to economists, the 20% tax imposed on foreign spends made by Indian citizens is
regressive in nature.

Q35.
Answer: d
Explanation:
● Securitization refers to an activity where a financier or lender transfers future receivables on a loan or
a bunch of loans to other financiers which helps with immediate liquidity requirements.
● Securitization pools or groups debt into portfolios. Issuers create marketable financial instruments by
merging various financial assets into tranches.
● Securitized instruments provide investors with income from interest and principal.
○ Mortgage-backed securities are backed by home loans issued to consumers.
○ Asset-backed securities are backed by auto loans, mobile home loans, credit card loans and
student loans.
Therefore, option (d) is the correct answer.
Relevance: According to a report, there has been a 60 per cent jump in securitization volumes to Rs 55,000 crore
in June quarter of FY 2023-24.

Q36.
Answer: a
Explanation:
● Dumping is a process where a company exports a product at a price lower than the price it normally
charges on its own home market. An anti-dumping duty is a protectionist tariff that a domestic
government imposes on foreign imports that it believes are priced below fair market value. Typically,
anti-dumping action means charging extra import duty on the particular product from the particular
exporting country in order to bring its price closer to the “normal value” or to remove the injury to
domestic industry in the importing country.
● The use of anti-dumping measures as an instrument of fair competition is permitted by the World Trade
Organisation (WTO). So, statement 1 is correct.
○ Disputes in the anti-dumping area are subject to binding dispute settlement before the Dispute
Settlement Body of the WTO.
● Anti-dumping and anti-subsidy duties are levied against exporter/country in as much as they are
country specific and exporter specific as against the customs duties which are general and universally
24
Vajiram & Ravi Prelims Test Series (2024)
Full Length Test – 05 – (R5534)
applicable to all imports irrespective of the country of origin and the exporter. So, statement 2 is not
correct.
○ Accordingly, there are variable rates of anti-dumping duty on different exporting countries,
producers or exporters.
Directorate General of Trade Remedies (DGTR):
● It is a quasi-judicial body functioning under the aegis of the Department of Commerce, Union Ministry
of Commerce and Industry, as an integrated single window agency for providing comprehensive and
swift trade defence mechanism in India.
● The DGTR conducts the anti-dumping/anti-subsidy & countervailing duty investigation and makes
recommendations to the Government for imposition of anti-dumping or anti-subsidy measures. Such
duty is finally imposed/levied by a notification of the Union Ministry of Finance. So, statement 3 is not
correct.
So, only one of the above statements is correct.
Therefore, option (a) is the correct answer.
Knowledge Box

Countervailing Duty (CVD):


● It is an additional import duty imposed on imported products (by the importing country)
when such products enjoy benefits like export subsidies and tax concessions in the
country of their origin.
● The objective of CVD is to nullify or eliminate the price advantage (low price) enjoyed
by an imported product when it is given subsidies or exempted from domestic taxes in
the country where they are manufactured.
● The WTO permits member countries to impose countervailing duty when the exporting
country gives an export subsidy.
Relevance: The Finance Ministry imposed definitive anti-dumping duty on ‘Wheel Loaders’ imports from China.

Q37.
Answer: a
Explanation:
● The Khudai Khidmatgar was a non-violent movement against British occupation of the Indian
subcontinent led by Abdul Ghaffar Khan, a Pashtun freedom fighter, in the North-West Frontier
Province. Following the arrest of Khan and other leaders in 1929, the movement formally joined the
Indian National Congress after they failed to receive support from the All-India Muslim League.
● Abdul Ghaffar Khan and other leaders of the Khudai Khidmatgar were arrested in 1930 by British police.
Khan’s arrest spurred protests in neighbouring towns, including Peshawar. Protests spilled into the
Qissa Khwani Bazaar in Peshawar on the day of Khan’s arrest. British soldiers entered the market area
to disperse crowds that had refused to leave. In response, British army vehicles drove into the crowds,
killing several protesters and bystanders. British soldiers then opened fire on unarmed protestors, killing
even more people.
Therefore, option (a) is the correct answer.

25
Vajiram & Ravi Prelims Test Series (2024)
Full Length Test – 05 – (R5534)
Q38.
Answer: b
Explanation:
Nagpur Session of Indian National Congress, 1920:
● It was held in the Nagpur district in Maharashtra and was presided over by C Vijayaraghavachariar. It
was during this session that the INC passed the main resolution on Non-Cooperation. So, statement 1 is
correct.
● It also appointed a fifteen-member Congress Working Committee, to guide the Congress. So, statement
2 is correct.
● An important change was made in this session. Now, instead of having the attainment of self-
government through constitutional means as its goal, the Congress decided to have the attainment of
swaraj through peaceful and legitimate means, thus committing itself to an extra-constitutional mass
struggle. So, statement 3 is not correct.
So, only two of the above statements are correct.
Therefore, option (b) is the correct answer.

Q39.
Answer: d
Description:
Karachi Resolution (1931):
● It was passed by the Indian National Congress (INC) at its 1931 Karachi session. It reiterated the
commitment of the INC to ‘Purna Swaraj’ or ‘complete independence’. In addition to Fundamental
Rights which protected civil liberties, the Resolution for the first time put forward a list of socio-
economic principles/rights that the Indian state had to adhere to.
● Under the Economic and Social Programme it mentioned the following:
○ With reference to labour, it mentioned about the protection of women workers and specially,
adequate provision for leave during maternity period. So, point 1 is correct.
○ With reference to taxation and expenditure, it mentioned that there shall be a drastic reduction
of military expenditure so as to bring it down to at least one-half of the present scale. So, point
2 is correct.
○ Relief of agricultural indebtedness, and control of usury – direct and indirect. So, point 3 is
correct.
○ Intoxicating drinks, and drugs shall be totally prohibited, except for medicinal purposes. So,
point 4 is correct.
○ The State shall own or control key industries and services, mineral resources, railways,
waterways, shipping and other means of transport. So, point 5 is correct.
○ The State shall provide for the Military training of citizens so as to organise a means of national
defence apart from the regular Military forces. So, point 6 is correct.
○ The State shall protect indigenous cloth; and for this purpose pursue the policy of exclusion of
foreign cloth and foreign yarn from the country and adopt such other measures as may be found
necessary. The State shall also protect other indigenous industries, when necessary, against
foreign competition.
26
Vajiram & Ravi Prelims Test Series (2024)
Full Length Test – 05 – (R5534)
○ Currency and exchange shall be regulated in the national interest.
So, all six of the above were part of the Karachi Resolution passed by the Indian National Congress in 1931.
Therefore, option (d) is the correct answer.

Q40.
Answer: b
Explanation:
● The concept of ‘martial race’ is a classification system that categorizes certain ethnic, religious, caste, or
social groups as being more suited for military service. ‘Non-martial races’ are considered unfit for
fighting. The British created the ‘martial race’ designation after the Indian Revolt of 1857. They
believed that ‘martial races’ were brave and well-built for fighting, while ‘non-martial races’ were unfit
for battle due to their sedentary lifestyles. So, statement 1 is correct.
● Martial Race Theory was ignited by the Peel Commission Report that contained the findings of the
enquiry into the causes of the Revolt of 1857. According to the report, British authorities lacked
knowledge about various Indian ethnic groups and their martial capabilities.
○ The report stated that the Revolt of 1857 began within the Bengal Army filled with high-caste
Brahmins, while groups like the Sikhs, Marathas, Rajputs and Gorkhas had upheld their loyalty
due to their understanding of honour and duty. These loyal groups were labelled as India’s
martial races. So, statement 2 is not correct and statement 3 is correct.
○ The ‘martial races’ of Punjab found disproportionately high representation in the newly
revamped Indian army.
So, only two of the above statements are correct.
Therefore, option (b) is the correct answer.

Q41.
Answer: d
Explanation:
● Mahatma Gandhi was associated largely with the Indian National Congress during the freedom struggle.
However, in 1934, Mahatma Gandhi had declared that he was severing all physical connections with
the Congress. On 17 September 1934, Mahatma Gandhi said that there was a growing and vital
difference of outlook between many Congressmen and himself.
○ One of the major differences was in the economic vision. Mahatma Gandhi wanted the
expansion of spinning and rural industries and wanted them to expand throughout the nook and
corner of India but was upset that only a few Congressmen endorsed the idea.
○ Another point of difference was the policy adopted towards untouchability between Mahatma
Gandhi and many of the Congressmen.
Therefore, option (d) is the correct answer.

Q42.
Answer: b
Explanation:

27
Vajiram & Ravi Prelims Test Series (2024)
Full Length Test – 05 – (R5534)
● Lord Dalhousie introduced a new system of internal communication in India. He is considered the father
of Indian Railways. Dalhousies’ famous Railway Minute of 1853 convinced the home authorities of the
need of the railways and laid down the main lines of their development. So, statement 1 is not correct.
● In 1849, the ‘guarantee system’ provided free land and a guaranteed five-percent rate of return to
private British companies willing to build railways. So, statement 2 is correct.
● Initially, around 1902, the railway network in India consisted of 19 privately owned separate railway
systems covering a total of 8,475 miles. Each company had formulated its own rules and regulations
regarding charging, booking and carriage of passengers and goods. Due to the multiplicity of rules, the
user was very much inconvenienced. With a view to overcome the difficulties in regard to different rules
and regulations and to frame rules regarding movement of wagons of one railway system over the other,
the Indian Railway Conference Association (IRCA) was formed in the year 1902. Initially the Association
was constituted to frame rules and regulations for booking of traffic and interchange of trains between
railways, to act as a consultative Committee and as a Board of Arbitration. So, statement 3 is correct.
So, only two of the above statements are correct.
Therefore, option (b) is the correct answer.

Q43.
Answer: c
Explanation:
● In 1777, a complaint was made by the directors of the English East India Company against the Supreme
Court as for them it was difficult to run the administration. To address this complaint, the British House
of Commons appointed a committee known as Touchet Committee to do an inquiry about the
administration of Bengal, Bihar, and Odisha.
● The report of this committee led to the enactment of the Act of Settlement of 1781.
Therefore, option (c) is the correct answer.

Q44.
Answer: c
Explanation:
● The Judicial Plan of 1772 was introduced by Warren Hastings. This plan consisted of regulations that
dealt with revenue and administration of civil and criminal justice.
● It divided the Diwani area of Bengal, Bihar, Orissa into several districts. As per the Judicial plan of 1772,
the divided districts were appointed with a collector in charge of the administration work. The company
servants were appointed as Collectors. These collectors were primarily responsible for revenue
collection and had direct control of revenue collection. So, statement 1 is correct.
● The Revenue Boards at Murshidabad & Patna were abolished & a supreme authority called the Board of
Revenue was set up at Calcutta which consisted of the Governor & all the members of his Council. So,
statement 2 is correct.
● Administration of Civil Justice Administration:
○ Mofussil Diwani Adalat:
■ Presided by the Collector and who acted as a Judge. So, statement 3 is correct.
■ Jurisdiction - all civil cases of revenue
28
Vajiram & Ravi Prelims Test Series (2024)
Full Length Test – 05 – (R5534)
○ Sadar Diwani Adalat:
■ Governor and two council members will act as a judge.
■ It used to have both appellate and original jurisdiction.
So, all three of the above statements are correct.
Therefore, option (c) is the correct answer.

Q45.
Answer: b
Explanation:
● The Chamber of Princes, also known as Narendra Mandal, was established in 1920 by a Royal
Proclamation of King Emperor George V to advocate the common interests of British India and the
princely states. Before the proclamation, an annual Princes’ Conference was held since 1916. So,
statement 1 is correct.
● The first meeting of the Chamber of Princes was held in 1921 and originally consisted of around 120
members. Moreover, a few of the major Indian princes also refused to participate and join the Chamber
of Princes. So, statement 2 is not correct.
● The Chamber of Princes recorded its insistent view that in future Constitution for the country, the
necessary assurance for safeguarding the autonomy and sovereignty of the various princely states and
for the protection of the rights and interests of the Indian princes. So, statement 3 is correct.
So, two of the above statements are correct.
Therefore, option (b) is the correct answer.

Q46.
Answer: a
Explanation:
● Pandurang Khankhoje was inspired by the freedom fighter Bal Gangadhar Tilak. He was also an ardent
admirer of the French Revolution and of the American War of Independence.
● He was one of the founding members of the Ghadar Party, established by Indians living abroad in 1914.
Its aim was to lead a revolutionary fight against the British in India.
● In 1907, Khankhoje and his friends in Japan established a secret Indian ‘Kranti Sena’. It consisted of a
small nucleus of young men studying the use of weapons and the rudiments of military training.
● He was revered as an agricultural scientist in Mexico. He was appointed a professor at the National
School of Agriculture in Chapingo, near Mexico City. He researched corn, wheat, pulses and rubber,
developing frost and drought-resistant varieties, and was part of efforts to bring in the Green Revolution
in Mexico.
Therefore, option (a) is the correct answer.

Q47.
Answer: b
Explanation:
● DNA polymorphism refers to the occurrence of two or more different sequences or forms of a specific
DNA segment within a population. These variations can arise from differences in a single nucleotide,
known as Single Nucleotide Polymorphisms (SNPs), or they can involve larger segments of the DNA.
29
Vajiram & Ravi Prelims Test Series (2024)
Full Length Test – 05 – (R5534)
● DNA polymorphisms are not necessarily associated with disease; they often represent the genetic
diversity within a population. This diversity is crucial for adaptation and evolution.
Therefore, option (b) is the correct answer.
Relevance: A recent study published in the International Journal of Obesity found that the gene polymorphism
and DNA methylation of GLP1R are both associated with metabolically healthy obesity (MHO).

Q48.
Answer: c
Explanation:
● Iron is an essential mineral in the human body, integral to various physiological processes, particularly
in oxygen transport and muscle function.
● Iron's primary role in the human body involves the formation of hemoglobin, a protein in red blood
cells responsible for carrying oxygen from the lungs to other parts of the body. It is also a key component
of myoglobin, a protein that supplies oxygen to muscles, crucial for muscular health and function. So,
statement 1 is correct.
● Hemochromatosis is a genetic condition where the body absorbs too much iron from food. This excess
iron accumulates in organs like the liver, heart, and pancreas, potentially leading to serious health issues.
If untreated, hemochromatosis can result in organ damage and other severe complications. This
condition illustrates the importance of iron regulation in the body. So, statement 2 is correct.
Therefore, option (c) is the correct answer.

Q49.
Answer: c
Explanation:
● Tissue culture is a technique for growing plant tissues isolated from the parent plant in an artificial
medium and controlled environment over a prolonged period under aseptic conditions. It is used on a
commercial scale in gerbera, orchids, bananas, carnation, anthurium, etc. It is based on the phenomenon
of the ‘totipotency’ of a cell, which denotes the capacity of a plant cell to regenerate into a full-fledged
plant having different organs. So, statement 1 is correct.
● Budding is the process of inserting a single mature scion bud into the stem (rootstock) in a way that
results in a union and continues to grow as a new plant. It is a type of grafting. It is an example of Asexual
propagation.
○ Asexual propagation is also called ‘vegetative propagation’. The vegetative parts of a plant like
a leaf, stem, root or their modified forms are used for propagation. Most horticultural crops are
commercially propagated by the vegetative or asexual method of propagation.
○ Plants propagated by asexual propagation are true-to-type genetically. This means that the
plants that are propagated asexually are genetically the same as the mother plant. So, statement
3 is correct.
○ By vegetative propagation, new varieties cannot be developed. So, statement 2 is correct.
So, all three of the above statements are correct.
Therefore, option (c) is the correct answer.

30
Vajiram & Ravi Prelims Test Series (2024)
Full Length Test – 05 – (R5534)
Q50.
Answer: d
Explanation:
● Induction cooking is a method of cooking that uses electromagnetic energy to cook food. It works on
the principle of electromagnetic induction. It uses an Alternating Current (AC) to generate an
electromagnetic field in a copper coil wire. This field creates an oscillating magnetic field that induces
eddy currents in ferromagnetic cookware. The eddy currents dissipate heat through the Joule effect,
which cooks food directly in the cookware. Some induction stoves can work with Direct Current (DC)
power supply as well.
● Induction cooktops use a magnetic field to generate heat, so they only work with cookware made of
ferrous metals. Ferrous metals contain iron. Some metal utensils that are compatible with induction
cooking include: cast iron, carbon steel and stainless steel etc. Thus, not all metal utensils can be used
in induction cooking. So, statement 1 is not correct and statement 2 is correct.
● Induction cooking is more energy efficient because there is no loss of heat in the transfer process.
Induction converts 90% of its energy to cooking heat, versus 40% for a gas stove.
So, Statement-I is incorrect but Statement-II is correct.
Therefore, option (d) is the correct answer.
Relevance: The National Efficient Cooking Programme (NECP) was recently launched to transform cooking
practices in India.

Q51.
Answer: a
Explanation:
● Muon radiography is a technique that uses muons to image the interior of large structures. It's similar
to X-ray radiography, which is used to image smaller objects. It works by measuring the absorption of
cosmic muons in matter. A muon detector tracks the muons and counts them.
● Muon radiography has several fascinating applications across various fields:
○ Archeology and Geology: It can be used to explore and visualize structures buried underground,
such as hidden chambers in pyramids, archaeological sites, or geological formations like caves
and faults. So, point 1 is correct.
○ Volcano and Mining Exploration: It enables the imaging of volcanic structures and the
assessment of potential hazards by mapping the internal structure of volcanoes. In mining, it aids
in visualizing and monitoring the structural integrity of mines. So, point 2 is correct.
○ Security and Border Control: It can be employed for non-invasive inspection of cargo containers,
detecting concealed materials or objects that might pose security threats. So, point 4 is correct.
○ Nuclear Waste Monitoring: It helps in monitoring nuclear waste storage facilities by detecting
changes or irregularities in the density of materials, ensuring the integrity of containment.
○ Civil Engineering and Infrastructure: It can assess the condition of infrastructure like bridges,
tunnels, and dams by examining their internal structures, identifying potential weaknesses or
damages without invasive measures.

31
Vajiram & Ravi Prelims Test Series (2024)
Full Length Test – 05 – (R5534)
● However, muon radiography isn't ideal for applications demanding ultra-high resolution, real-time
imaging, medical diagnostics, molecular analysis, detailed material composition analysis, or extremely
precise measurements. So, points 3 and 5 are not correct.
Therefore, option (a) is the correct answer.
Knowledge Box

Muons:
● They are subatomic particles that are created when cosmic rays collide with
particles in Earth's atmosphere.
● They are similar to electrons but are about 200 times heavier.
● Muons are highly penetrative and can be either positively or negatively charged.
Relevance: Egyptian officials announced the discovery of a hidden corridor inside the Great Pyramid of Giza
using cosmic-ray muon radiography.

Q52.
Answer: d
Explanation:
● In biology, a pathogen in the oldest and broadest sense, is any organism or agent that can produce
disease.
● Cryptococcus neoformans (C. neoformans) is a globally distributed pathogenic yeast (fungus) which
lives in the environment (soil, decaying wood). After inhalation of fungal cells from the environment, C.
neoformans can infect humans causing Cryptococcosis. It initially affects the lungs but can spread to the
central nervous system (cryptococcal meningitis) and blood (cryptococcaemia).
● Aspergillus fumigatus is a ubiquitous environmental mould that can infect humans and cause
aspergillosis. It is inhaled from the environment, predominantly causing pulmonary disease, but can
disseminate to other sites, such as the brain. Preventability is high but the vaccine is not available.
Therefore, option (d) is the correct answer.
Relevance: An international team of researchers has unveiled ground-breaking findings on Aspergillus
fumigatus, which can cause deadly disease in humans.

Q53.
Answer: a
Explanation:
● Sodium and potassium are the only elements that can be observed through telescopes in the lunar
atmosphere (exosphere). It is due to the following reasons:
○ Sunlight resonance: Sodium and potassium have specific wavelengths that "glow" under the
Sun's radiation, making them easier to detect. Other elements lack this special interaction with
sunlight.
○ Vaporization: The Moon's harsh environment allows these elements to exist as vapor in the very
thin atmosphere, while most others stay bound to the surface. So, point 1 and point 3 are
correct.

32
Vajiram & Ravi Prelims Test Series (2024)
Full Length Test – 05 – (R5534)
● Helium and Neon cannot be observed through a telescope in the lunar atmosphere. So, points 2 and 4
are not correct.
Therefore, option (a) is the correct answer.
Knowledge Box

● The Moon, unlike Earth, does not have a substantial atmosphere. The moon's
atmosphere is very thin compared to Earth's atmosphere.
● The Moon's atmosphere is made up of: Helium (33%), Neon (33%), Hydrogen (33%),
Argon (20,000–100,000 atoms per cubic centimeter), Sodium (70 atoms per cubic
centimeter) and Potassium (17 atoms per cubic centimeter).
● The Moon's atmosphere also contains trace amounts of ammonia, methane,
potassium, sodium and carbon dioxide.
● The Moon's atmosphere is not made up of a gaseous mixture that could support
oxygen-dependent mammals like humans.
Relevance: Scientists from the Indian Space Research Organisation (ISRO) have mapped out the global
distribution of sodium on the Moon’s surface.

Q54.
Answer: c
Explanation:
Internet Leased Line:
● It is an exclusive, allocated and dedicated data connection or internet with a fixed bandwidth. It is an
allocated circuit between two points of communication. It provides services beyond what standard
broadband offers, such as uncontended symmetrical speeds for uploads and downloads.
○ It enables data-driven businesses to connect to the internet in a secure, reliable and highly
efficient manner, with maximum upload and download speeds, resilience and uptime. Leased
lines often have speeds ranging from 1 Mbps to 10 Gbps.
● It is leased directly from an Internet Service Provider (ISP) to a business, typically for a monthly rental
charge or according to the terms agreed upon with the service provider.
○ The types of the leased lines include Fibre leased lines, Digital Subscriber Line (DSL) leased lines,
Multi-Protocol Label Switching (MPSL) leased lines.
Therefore, option (c) is the correct answer.
Knowledge Box

Difference between Leased Line and Broadband:

Leased Line Broadband

Downloads and uploads have identical Download speed is more than upload speed
speeds

33
Vajiram & Ravi Prelims Test Series (2024)
Full Length Test – 05 – (R5534)
Speeds are extremely high in this setup Speeds are moderate in this, especially as
compared to Leased line

The connection here is dedicated, does not Bandwidth is shared between several
cause interruptions people

Q55.
Answer: c
Explanation:
Hard fork:
● It relates to blockchain technology and is a radical change to a network's protocol that makes previously
invalid blocks and transactions valid, or vice-versa. A hard fork requires all nodes or users to upgrade
to the latest version of the protocol software.
● It refers to a radical change to the protocol of a blockchain network that effectively results in two
branches, one that follows the previous protocol and one that follows the new version.
● Here, holders of tokens in the original blockchain will be granted tokens in the new fork as well, but
miners must choose which blockchain to continue verifying.
● It can occur in any blockchain, and not only Bitcoin (where hard forks have created Bitcoin Cash and
Bitcoin SV, among several others, for example).
Therefore, option (c) is the correct answer.
Knowledge Box

Soft Fork:
● In blockchain technology, a soft fork is a change to the software protocol where only
previously valid transaction blocks are made invalid. Because old nodes will recognize
the new blocks as valid, a soft fork is backwards-compatible. This kind of fork requires
only a majority of the miners upgrading to enforce the new rules, as opposed to a hard
fork that requires all nodes to upgrade and agree on the new version.

Q56.
Answer: b
Explanation:
● Prisoners do not have voting rights in India. But this restriction does not apply to a person subjected to
any kind of preventive detention. The Representation of the People Act (RPA), 1951 confers voting
rights on the electors subjected to preventive detention. As per Rule 18 of the Conduct of Elections
Rules, 1961, the electors under preventive detention are entitled to cast their votes by post. So, point 1
is not correct.
● The RPA, 1951 imposes a blanket ban on under trials, persons confined in civil prisons and convicts
serving their sentence in jails, from casting their votes. The Act mandates that “no person shall vote at
any election if he is confined in a prison, whether under a sentence of imprisonment or transportation
or otherwise, or is in the lawful custody of the police”. So, point 2 is correct.
34
Vajiram & Ravi Prelims Test Series (2024)
Full Length Test – 05 – (R5534)
● Till the year 2010, Non-Resident Indians (NRIs) were not allowed to vote in the elections. However, post
2010 an amendment permitted NRIs to vote at their particular constituency in India but the voting
required the direct presence of the NRI in India at the particular polling booth where the name was
registered. So, point 3 is not correct.
Therefore, option (b) is the correct answer.

Q57.
Answer: c
Explanation:
● Cigarettes and Other Tobacco Products Act, 2004:
○ It states that no person shall smoke in any public place; provided that in a hotel having thirty
rooms or a restaurant having seating capacity of thirty persons or more and in the airports, a
separate provision for smoking area or space may be made. So, statement 1 is correct.
○ It states that no person shall take part in any advertisement which directly or indirectly suggests
or promotes the use or consumption of cigarettes or any other tobacco products. So, statement
2 is correct.
● It is mandatory for Over-The-Top (OTT) streaming platforms to display anti-tobacco warnings, as per
the notification of the Union Ministry of Health and Family Affairs. They shall also be required to exhibit
an anti-tobacco health warning as a prominent static message at the bottom of the screen when tobacco
products or their use are displayed during the programme. So, statement 3 is correct.
So, all three of the above statements are correct.
Therefore, option (c) is the correct answer.

Q58.
Answer: c
Explanation:
● Article 301 of the Constitution of India provides that the trade, commerce and intercourse in the country
should be free throughout the country. So, statement 1 is correct.
● Under Article 302, the Parliament can restrict the freedom of trade between different States, if it is
necessary for the public interest. This restriction can be applied to any State or it may also be placed in
any part of the territory of India. So, statement 2 is correct.
● Article 303 (1) of the Constitution of India states that the Parliament cannot impose a restriction on any
State in favour of another State. It means that no discriminatory restriction can be made by the
Parliament which gives benefit to one State while the other States are excluded from such benefit.
○ Clause (2) of the Article 303 engrafts an exception to the limitation contained in clause (1), in as
much as it permits Parliament to make a law giving preference, or making discrimination
between one State and another, if it is declared by such law that it is necessary to do so for the
purpose of dealing with a situation arising from scarcity of goods in any part of the territory of
India. So, statement 3 is correct.
So, all three of the above statements are correct.
Therefore, option (c) is the correct answer.

35
Vajiram & Ravi Prelims Test Series (2024)
Full Length Test – 05 – (R5534)
Q59.
Answer: a
Explanation:
● In 1975, the first Cricket World Cup was contested in England as a series of one-day matches of 60 overs
per side. It was held in England. So, statement 1 is not correct.
● India has hosted the Cricket World Cup in the years 1987, 1996, 2011 and 2023. The Cricket World Cup
for 2011 was held in Bangladesh, India and Sri Lanka. So, statement 2 is not correct.
● The Australian cricket team is the only country to have won 5 world cup titles until now. They are also
the only team to have won three Cricket World Cups in a row - 1999, 2003 and 2007. So, statement 3
is correct.
● India has won only 2 World Cups, in 1983 and 2011. India reached finals in 2003 and 2023. However, in
both the years India was defeated by Australia.
So, only one of the above statements is correct.
Therefore, option (a) is the correct answer.

Q60.
Answer: c
Explanation:
● The European Commission describes “temporary protection” under the Temporary Protection Directive
(TPD) of 2001 as an “exceptional measure to provide immediate and temporary protection to displaced
persons from non-European Union (EU) countries and those unable to return to their country of origin”.
● It reduces disparities between the policies of European Union (EU) States on the reception and
treatment of displaced persons in a situation of mass influx. It also promotes solidarity and burden-
sharing among EU States with respect to receiving large numbers of potential refugees at one time.
● Recently, amid the Russia-Ukraine war, EU Member States made the unprecedented decision to activate
the European Union’s Council Directive 2001/55/EC of 20 July 2001, known as the Temporary Protection
Directive (TPD).
Therefore, option (c) is the correct answer.

Q61.
Answer: c
Explanation:
● The DUS test, consisting of Distinctiveness, Uniformity, and Stability, is a crucial component of the
Protection of Plant Varieties and Farmers’ Rights Act, 2001. It is employed to assess the distinct
characteristics, consistent performance and stability of a plant variety over generations.
● The objective is to grant intellectual property protection, such as Plant Breeders' Rights, to plant
varieties that meet these criteria. It ensures that the protected varieties are unique, uniform and stable,
promoting innovation and protecting the rights of plant breeders.
Therefore, option (c) is the correct answer.
Relevance: The Delhi High Court has held that Section 19 of the Protection of Plant Variety Act (PPV Test)
mandates the completion of the DUS Test before the advertisement of a plant variety application.

36
Vajiram & Ravi Prelims Test Series (2024)
Full Length Test – 05 – (R5534)
Q62.
Answer: d
Explanation:
● The ‘Paris Principles’ (Principles Relating to the Status of National Human Rights Institutions (NHRIs)) set
out the minimum standards that NHRIs must meet in order to be considered credible and to operate
effectively. The key pillars of the Paris Principles are pluralism, independence and effectiveness. The
NHRIs shall have an infrastructure which is suited to the smooth conduct of its activities, in particular
adequate funding. The purpose of this funding should be to enable it to have its own staff and premises,
in order to be independent of the Government and not be subject to financial control. So, statement 1
is correct.
● ‘Declaration on Human Rights Defenders’ was adopted by consensus by the United Nations General
Assembly in 1998, on the occasion of the fiftieth anniversary of the Universal Declaration of Human
Rights. It identifies human rights defenders as individuals or groups who act to promote, protect or strive
for the protection and realisation of human rights and fundamental freedoms through peaceful means.
It is not a legally binding instrument. So, statement 2 is correct.
● The ‘Universal Declaration of Human Rights (UDHR)’ was proclaimed by the United Nations General
Assembly in 1948 as a common standard of achievements for all peoples and all nations. It sets out, for
the first time, fundamental human rights to be universally protected. It says that everyone has the right
to rest and leisure, including reasonable limitations of working hours and periodic holidays with pay.
So, statement 3 is correct.
Therefore, option (d) is the correct answer.

Q63.
Answer: a
Explanation:
● Fungi can be single celled or very complex multicellular organisms. They are found in just about any
habitat but most live on the land, mainly in soil or on plant material rather than in sea or freshwater. So,
statement 1 is not correct.
○ A group called the decomposers grow in the soil or on dead plant matter where they play an
important role in the cycling of carbon and other elements.
● Some are parasites of plants causing diseases such as mildews, rusts, scabs or canker. A very small
number of fungi cause diseases in animals. In humans these include skin diseases such as athletes’ foot,
ringworm and thrush. So, statement 2 is correct.
Yeasts:
● They are a type of fungi, small, lemon-shaped single cells that are about the same size as red blood cells.
They multiply by budding a daughter cell off from the original parent cell.
● Yeasts such as Saccharomyces play an important role in the production of bread and in brewing. Other
species of yeast such as Candida are opportunistic pathogens and cause infections in individuals who do
not have a healthy immune system. So, statement 3 is not correct.
● Yeasts are also one of the most widely used model organisms for genetic studies, for example in cancer
research.
So, only one of the above statements is correct.
37
Vajiram & Ravi Prelims Test Series (2024)
Full Length Test – 05 – (R5534)
Therefore, option (a) is the correct answer.

Q64.
Answer: a
Explanation:
● The operational carbon footprint of a building is the sum of all the carbon produced over the lifetime
use of the building, which could easily be 50 or more years. This includes greenhouse gas emissions from
building energy consumption, such as heat, lighting and ventilation.
● Operational carbon is associated with the everyday running of a building. Embodied carbon is the
carbon footprint of a building before it is built, and encompasses the greenhouse gases emitted during
the construction process. This includes greenhouse gas emissions from the manufacturing,
transportation, installation, maintenance and disposal of building materials. Embodied carbon is emitted
once and cannot be reduced after a building is constructed or renovated. So, statement 1 is correct.
● Operational carbon footprint includes actions like lighting, heating, ventilation, cooling, or air
conditioning. So statement 2 is not correct.
● Embodied carbon footprint includes things like extraction and production of materials, transportation
of materials and manufacturing. So statement 3 is not correct.
So, only one of the above statements is correct.
Therefore, option (a) is the correct answer.

Q65.
Answer: c
Explanation:
● The Public Liability Insurance Act, 1991, guarantees relief to victims of accidents in hazardous
industries and imprisonment as penal action against owners. So, statement 1 is correct.
● As per the recent public notice issued by the Union Ministry of Environment, Forest and Climate Change
(MoEF&CC), the Act will be amended to remove the provision that entails the prosecution of owners
in cases of industrial accidents.
○ The aforementioned amendment has been justified by the Government on the grounds that
elimination of the penal provisions under the PLI Act, 1991, will ameliorate fear of imprisonment
for minor lapses such as delay or incorrect filing of a compliance report.
○ While violations of the Act will not attract prosecution, non-payment of penalty will be
considered as a penal offence, as per the proposed amendments.
● Grievances pertaining to non-payment of penalty can be taken to the National Green Tribunal. So,
statement 2 is correct.
● Further, while provisions will be made for the imposition of “substantial” penalties, the limits of
insurance policies, which owners of hazardous industries have to mandatorily obtain for workers in
accordance with the Act, have been increased substantially.
Environment Relief Fund (ERF):
● Instituted under the Public Liability Insurance Act, 1991 (PLIA) in the wake of the Bhopal Gas Tragedy,
its primary purpose is to offer immediate relief to victims of accidents involving hazardous substances.
So, statement 3 is correct.
38
Vajiram & Ravi Prelims Test Series (2024)
Full Length Test – 05 – (R5534)
So, all three of the above statements are correct.
Therefore, option (c) is the correct answer.
Relevance: The Union Ministry of Environment, Forest and Climate Change (MoEF&CC) issued a public notice
decriminalising the Public Liability Insurance Act, 1991.

Q66.
Answer: b
Explanation:
● Dryland Farming refers to the cultivation of crops under natural rainfall conditions without or very
scanty irrigation. Dryland areas are characterised by low rainfall within a range of 375 mm to 1125 mm,
which are unevenly distributed, highly erratic, and uncertain. These areas have generally poor or
degraded soils with low water holding capacities and multiple nutrient deficiencies.
● There is a vast scope to increase the productivity of dryland agriculture from the current average of 1.2
tonnes per hectare to 2.0 tonnes per hectare. This can be done through following manner:
● Mulching is a common dryland technique to conserve moisture in the soil by preventing evaporation.
Mulch is a material, generally straw, leaves, or plastic, that is spread over the soil's surface to prevent its
natural exposure to sunlight. Mulch also helps to keep the roots of plants cooler, which can help them
survive during periods of drought. So, point 1 is correct.
● Shelterbelts and windbreaks are other common water conservation techniques prevalent in dryland
farming. These are rows of trees and shrubs planted on the ridge or around the field to provide wind.
They not only help reduce evaporation but also protect crops from damage by strong winds. So, point 2
is correct.
● Deep tillage and contour ploughing allow water to penetrate deeper into the ground. Check dams and
farm ponds are common water harvesting structures in rainfed regions that help provide life-saving
irrigation during long dry spells. So, point 3 is correct.
● Rice and sugarcane are both water-intensive crops. This should not be grown in dryland farming. They
require a huge quantity of water for suitable growth. So, point 4 is not correct.
● Weed control is an important strategy in dryland farming because, if left unchecked, weeds can compete
with crops for water.
● Crop rotation in the field on a yearly basis helps prevent soil depletion and maintain fertility.
Therefore, option (b) is the correct answer.

Q67.
Answer: d
Explanation:
● Scientists have recently discovered a new fish species from the Milak river, Badis limaakumi, in
Mokokchung district of Nagaland. It belongs to the family of Badidae, a small freshwater fish found in
streams with slow or moderate water flow. These fish are known as chameleon fish for their ability to
change colour. This helps them blend with the surroundings when under stress.
● Apart from channels of rivers, the edible fish are found in ditches and stagnant water bodies across
India, Bangladesh, Nepal, Pakistan, Thailand and Myanmar.
Therefore, option (d) is the correct answer.
39
Vajiram & Ravi Prelims Test Series (2024)
Full Length Test – 05 – (R5534)
Relevance: Badis limaakumi is a new species of freshwater ray-finned fish from Nagaland, India.

Q68.
Answer: c
Explanation:
● Chlorophyll is a crucial pigment for photosynthesis in plants, with various types playing unique roles in
capturing sunlight and converting it into energy.
○ Chlorophyll a is the primary pigment involved in photosynthesis, found in all photosynthetic
organisms, from higher plants to algae. It absorbs light at specific wavelengths, particularly in the
blue and red regions, initiating the light-dependent reactions of photosynthesis. So, statement 1
is correct.
○ Chlorophyll c is typically found in some algae and protists, rather than in higher plants. It aids in
light harvesting but does not broaden the light spectrum used in photosynthesis like chlorophyll
b. Chlorophyll b, not c, is the accessory pigment in higher plants that broadens the spectrum of
light usable for photosynthesis by absorbing light at slightly different wavelengths compared to
chlorophyll a. So, statement 2 is not correct.
● Chlorophyll d is a unique pigment mainly found in red algae. It has distinctive light absorption
properties, absorbing far-red light, which is at the edge of the visible spectrum and beyond what
chlorophyll a can absorb efficiently. This rare pigment adapts red algae to deeper water environments
where far-red light predominates. So, statement 3 is correct.
Therefore, option (c) is the correct answer.

Q69.
Answer: d
Explanation:
● The Global Survey on Digital and Sustainable Trade Facilitation conducted by United Nations Economic
and Social Commission for Asia Pacific (UNESCAP) covers World Trade Organisaion (WTO) Trade
Facilitation Agreement (TFA) along with set of nearly 60 trade facilitation measures categorized into
eleven sub-groups, namely: Transparency; Formalities; Institutional arrangement and cooperation;
Transit facilitation; Paperless trade; Cross-border paperless trade; Trade facilitation for SMEs;
Agricultural trade facilitation; Women in trade facilitation; Trade finance for trade facilitation; and Trade
facilitation in times of crisis.
● The 2023 survey, covering more than 140 economies and evaluating 60 trade facilitation measures, has
positioned India at the forefront of global trade facilitation efforts, with an impressive score of 93.55%
in 2023 vis a vis 90.32% in 2021.
○ India continues to demonstrate its commitment to digital and sustainable trade facilitation, as
evidenced by its outstanding performance in the survey.
○ The 2023 Survey has recognized India's exceptional progress across various sub indicators, with
the country achieving a perfect score of 100% in four key areas: Transparency, Formalities,
Institutional Arrangement and Cooperation and Paperless Trade.

40
Vajiram & Ravi Prelims Test Series (2024)
Full Length Test – 05 – (R5534)
○ India is now the best performing country amongst all the countries of the South Asia region.
The overall score of India has been greater than many developed countries including Canada,
France, UK, Germany etc.
Therefore, option (d) is the correct answer.
Relevance: India has demonstrated exceptional performance in the Global Survey on Digital and Sustainable
Trade Facilitation conducted by the United Nations Economic and Social Commission for Asia Pacific (UNESCAP).

Q70.
Answer: b
Explanation:
Mammatus clouds:
● They are some of the most unusual and distinctive cloud
formations with a series of bulges or pouches emerging from
the base of a cloud.
● The shape of mammatus formations can vary widely; from
the classic protruding shape, to a more elongated tube
hanging from the cloud above.
Formation of mammatus clouds:
● Typically, turbulence within the cumulonimbus cloud will
cause mammatus to form, especially on the underside of the
projecting anvil as it rapidly descends to lower levels.
Weather associated with mammatus clouds:
● Mammatus clouds generally form in the most unstable cumulonimbus, meaning that there is also a
chance of hail, heavy rain and lightning in the vicinity and if the air is cold enough during winter they can
produce snow.
● Sometimes mammatus may form on other cloud types which produce no rain, though this is far less
common.
Therefore, option (b) is the correct answer.
Relevance: The city of Istanbul was recently engulfed in darkness during the day when mammatus clouds
covered the sun for almost five minutes.

Q71.
Answer: b
Explanation:
● Seismic Zonation map of India (state wise) given by Bureau of Indian Standards (BIS), Ministry of
Consumer Affairs, Food & Public Distribution, Government of India, has categorised the country into
several seismic zones (Zone II to Zone V). So, statement 1 is not correct.
● The seismic zones of the entire country are classified based upon the empirical seismic attenuation law
with respect to the maximum credible earthquake source zone located at the vicinity of the area. It is
further corroborated with historical seismicity and ground motions of the earthquakes observed in
various parts of the country in the past. So, statement 2 is correct.
● The Northeastern part of India is the most earthquake-prone region and is placed in zone V. So,
statement 3 is correct.
41
Vajiram & Ravi Prelims Test Series (2024)
Full Length Test – 05 – (R5534)
○ The latest seismic zone map of India was released in 2002 with only four zones – II, III, IV, and V.
Approximately 11% of the country falls in zone V,
18% in zone IV, 30% in zone III and the remaining
in zone II.
● According to the revised IS 1983-2002 seismic code, zones
are mapped to a modified Modified Mercalli Intensity
(MMI) scale with a Comprehensive Intensity Scale (CIS-64),
an alternative to the MMI scale for seismic zoning.
○ The MMI scale takes into account the effect of
earthquakes on people, objects, and buildings, and
estimates the shaking intensity from an
earthquake at a specific location.
So, only two of the above statements are correct.
Therefore, option (b) is the correct answer.
Knowledge Box

● National Centre for Seismology under Ministry of Earth Sciences is the nodal agency of
Government of India (GoI), for monitoring earthquakes in and around the country.
○ For this purpose, NCS maintains a National Seismological Network (NSN)
consisting of 115 observatories spread across the country.
Relevance: Hundreds of houses in Joshimath, a town in Uttarakhand located in Zone V of India’s seismic zonation
scheme, developed cracks.

Q72.
Answer: c
Explanation:
● A fjord is a deep, narrow and elongated sea or lake drain, with steep land on three sides. The opening
toward the sea is called the mouth of the fjord, and is often shallow. So, statement 1 is correct.
● The fjords were formed by the giant glacier tongues that through several ice ages have shaped the
landscape. A fjord is thus a U-shaped undersea valley often surrounded by dramatic mountain scenery.
So, statement 2 is correct.
● Some of the largest coral reefs are found at the bottom of fjords in Norway. They are home to several
types of fish, plankton and sea anemones. Scientists know much less about these deep, cold-water
reefs. So, statement 3 is not correct.
● Skerries are also found around fjords. A skerry is a small, rocky island created through glaciation. Most
of the Scandinavian coastline is cut into thousands of little blocks of land. So, statement 4 is correct.
So, only three of the above statements are correct.
Therefore, option (c) is the correct answer.

Q73.
Answer: c
Explanation:
42
Vajiram & Ravi Prelims Test Series (2024)
Full Length Test – 05 – (R5534)
● A radiation belt refers to a layer of charged and energetic particles which is held by the planet’s magnetic
field around the planet.
Van Allen Radiation belt:
● It specifically refers to the radiation belts around the Earth. They were discovered by a group of scientists
in 1958 under the supervision of Dr James Van Allen. The Earth has two belts, one is Van Allen and the
second appears to be temporary. These belts are found in the inner part of the Earth's magnetosphere.
● The main constituents of the belts are believed to come from solar wind and cosmic rays. The belt is
divided into two belts - outer and inner. The inner belt extends from roughly 600 miles to 3,700 miles
above the earth's surface. The outer belt extends from 9,200 miles to 15,600 miles above the surface.
○ The outer belt of the radiation belt is formed by the energetic and charged electrons and the
inner belt contains a combination of protons and electrons and is formed as a result of cosmic ray
collisions in the upper atmosphere. These particles do not have enough energy to escape into
space.
● Planets like Jupiter and Saturn have similar belts. The Sun itself has only temporary radiation belts
because it lacks a global dipole field.
Therefore, option (c) is the correct answer.

Q74.
Answer: d
Explanation:
● A playa is a dry, vegetation-free, flat area at the lowest part of an undrained desert basin. It is a location
where ephemeral lakes form during wet periods, and is underlain by stratified clay, silt, and sand, and
commonly, soluble salts. Playas are generally covered in fine-grained material that may or may not
display mudcracks. Playas may also contain evaporite minerals such as salt or gypsum. So, statement 1
is correct.
● Enclosed basins of salt and clay accumulation may originate from numerous causes. Tectonic causes
include faulting, as in the East African Rift Valley and Death Valley, and warping, as in Lake Eyre in
Australia. So, statement 2 is correct.
● The Sambhar Lake is a playa located at the eastern fringe of the Thar Desert in Rajasthan. For more than
four decades, this lake has attracted enormous attention due to its hypersalinity. The hypotheses to
explain the geological evolution of the Sambhar have ranged from marine connection, river blockage by
dunes and tectonics. So, statement 3 is correct.
Therefore, option (d) is the correct answer.

Q75.
Answer: a
Explanation:
● Recently, the Union Ministry of Textiles announced the “Kasturi Cotton Bharat” brand of cotton by which
Indian cotton has been endowed with a brand and a logo that represents whiteness, softness, purity,
lustre and Indianness.
● The Kasturi Cotton Bharat is a joint initiative by the Ministry of Textiles, the Cotton Corporation of
India, Trade Bodies & Industry to work on the principle of self-regulation by owning complete
43
Vajiram & Ravi Prelims Test Series (2024)
Full Length Test – 05 – (R5534)
responsibility of Branding, Traceability and Certification of Indian Cotton to enhance its
competitiveness in the global market and create a sustainable ecosystem for all stakeholders involved.
● All the ginners in the country have been empowered to produce Kasturi Cotton Bharat brand as per
stipulated protocol. Besides this, to provide complete traceability of Kasturi Cotton Bharat across the
supply chain, QR based certification technology will be used at each stage of the processing and a
blockchain based software platform will provide end to end traceability and transaction certificate.
Therefore, option (a) is the correct answer.
Relevance: The Union Minister of Textiles recently launched the Kasturi Cotton Bharat website.

Q76.
Answer: b
Explanation:
● The Indian capital market can be classified into two major segments, the primary market and the
secondary market. Although they work very differently, the main objective of these two types of markets
is the same, to act as platforms for the exchange or trade of financial securities between buyers and
sellers. Financial securities like stocks, bonds, debentures and mutual funds are traded in the primary
and secondary markets.
● The primary market represents a platform where securities such as equity shares, bonds, and
debentures are issued to the general public for the first time. The exchange of securities in a primary
market happens directly between the investors and the company issuing the securities. So, statement
1 is correct.
● The secondary market is a platform where securities previously issued via the primary market are traded
freely between investors. Here, the exchange of securities happens between investors without the
involvement of the company or the issuing entity.
○ Since trading on the secondary market happens between investors, the issuing company is not
impacted in any way. Any change in the share price due to secondary market trading neither
affects the financial situation nor the capital structure of the issuing company. So, statement 2
is not correct.
● In a primary market, the price of securities is fixed by the company issuing the securities. Whereas, the
forces of demand and supply primarily determine the price of a security in the secondary market. So,
statement 3 is not correct.
● Unlike the primary market, where the securities are open for subscription only for a short period,
trading in the secondary market happens continuously. Interested investors can purchase and sell
securities anytime during the market hours. So, statement 4 is correct.
So, only two of the above statements are correct.
Therefore, option (b) is the correct answer.
Relevance: There has been a boom in the primary market, whereas the secondary market has been slogging.

Q77.
Answer: c
Explanation: The International Labour Organization (ILO) defines unemployment as being out of a job, being
available to take a job and actively engaged in searching for work. The labour force is defined as the sum of the
44
Vajiram & Ravi Prelims Test Series (2024)
Full Length Test – 05 – (R5534)
employed and the unemployed. The unemployment rate is measured as the ratio of the unemployed to the
labour force.
● The Government of India relies on the Periodic Labour Force Survey (PLFS) to calculate the
unemployment rate or the ratio of persons unemployed in the labour force. It is conducted by the
National Statistical Office (NSO) under the Union Ministry of Statistics and Programme
Implementation. PLFS was designed with two major objectives for measurement of employment and
unemployment. It measures unemployment in both urban and rural areas by estimating key parameters
in both ‘Usual Status’ (Principal Status + Subsidiary Status) and ‘Current Weekly Status’ (CWS). So,
statements 1 is not correct and statement 3 is correct.
● The Labour Bureau has been entrusted with the task of conducting the All-India Quarterly
Establishment based Employment Survey (AQEES). AQEES has two components namely Quarterly
Employment Survey (QES) in respect of establishments employing 10 or more workers (mostly
constituting the ‘organised’ segment) and Area Frame Establishment Survey (AFES) to build up a frame
in respect of establishments employing 9 or less workers (mostly the ‘unorganised’ segment). AQEES
provides estimates of employment, vacancies, training and other related parameters for major non-farm
sectors of the economy. So, statement 2 is not correct.
Therefore, option (c) is the correct answer.
Relevance: The Periodic Labour Force Survey (PLFS) has been released by the National Statistical Office (NSO).

Q78.
Answer: b
Explanation:
● Multi Level Marketing (MLM)/Chain Marketing/Pyramid Structure and Money Circulation Schemes
promise easy or quick money upon enrolment of members. Income under such schemes majorly comes
from enrolling more and more members from whom hefty subscription fees are taken rather than from
the sale of products they offer. All these schemes are cognizable offences under the Prize Chit and
Money Circulation (Banning) Act 1978. So, points 1 and 2 are correct.
● Algorithmic trading or algo trading monitors live stock prices and executes a trade upon fulfilment of a
specific criteria. Securities and Exchange Board of India (SEBI) introduced algorithmic trading in India in
2008. So, point 3 is not correct.
So, only two of the above are legally banned in India.
Therefore, option (b) is the correct answer.
Relevance: Recently, the Government and the Central Bank in India stepped up to ensure awareness through
greater financial literacy, while dealing with fraudulent activities.

Q79.
Answer: c
Explanation:
● Article 115 of the Constitution of India provides for supplementary, additional or excess grants.
Demand for grants are estimates of expenditure from the Consolidated Fund of India that are included
in the annual financial statement (Budget) and must be voted on in the Lok Sabha. These grants are

45
Vajiram & Ravi Prelims Test Series (2024)
Full Length Test – 05 – (R5534)
submitted in pursuance of Article 113 (Procedure in Parliament with respect to estimates) of the
Constitution.
● Token grants are sought when funds to meet proposed expenditure on a new service can be made
available by reappropriation from the savings under other grants, and the amount required is less than
Rs 100. This grant is more of a procedural requirement to comply with the rules that no amount can be
spent without the Parliament's approval. So, statement 1 is correct.
● Excess Grant is granted when money has been spent on any service during a financial year in excess of
the amount granted for that service in the budget for that year. It is voted by the Lok Sabha after the
financial year and members can point out how money has been spent unnecessarily.
○ Before the demands for excess grants are submitted to the Lok Sabha for voting, they must be
approved by the Public Accounts Committee of Parliament. So, statement 2 is correct.
Therefore, option (c) is the correct answer.
Knowledge Box
● Additional Grant is granted when a need has arisen during the current financial
year for additional expenditure upon some new service not contemplated in the
budget for that year.
● Supplementary Grant is granted when the amount authorised by the Parliament
through the appropriation act for a particular service for the current financial year
is found to be insufficient for that year. The Supplementary grants are presented
to and passed by the House before the end of the financial year and no discussion
can be raised on the original grants.
Relevance: The Parliament recently approved the Central Government’s second and final batch of
supplementary demands for grants for the FY 2022-23, amounting to Rs 1.48 lakh crore.

Q80.
Answer: d
Explanation:
● An asset class is a grouping of investments that exhibit similar characteristics and are subject to the
same laws and regulations. There is usually very little correlation and sometimes a negative correlation
among different asset classes. Financial advisors focus on asset classes as a way to help investors
diversify their portfolios.
● Equities (e.g., stocks), fixed income (e.g., bonds), cash and cash equivalents, real estate, commodities
and currencies are common examples of asset classes. Each asset class is expected to reflect different
risk and return investment characteristics and perform differently in any given market environment. So,
points 1, 2 and 4 are correct.
● Sovereign Gold Bonds are also examples of asset classes in India. So, point 3 is correct.
So, all four of the above are categorised as ‘asset classes’ in India.
Therefore, option (d) is the correct answer.

46
Vajiram & Ravi Prelims Test Series (2024)
Full Length Test – 05 – (R5534)
Q81.
Answer: a
Explanation:
● The National Company Law Tribunal (NCLT) is a quasi-judicial authority incorporated in 2016 for dealing
with corporate disputes that are of civil nature arising under the Companies Act, 2013. So, statement
1 is not correct.
○ It was established based on the recommendation of the Balakrishna Eradi committee on law
relating to the insolvency and the winding up of companies. It shall consist of a President and
such number of judicial and technical members as may be required.
● It is empowered with adjudicating authority relating to Indian companies including proceedings relating
to arbitration, compromise, arrangements, reconstructions and the winding up of companies, insolvency
resolution process of companies and limited liability partnerships under the Insolvency and Bankruptcy
Code, 2016. So, statement 2 is correct.
● The National Company Law Appellate Tribunal (NCLAT) was constituted for hearing appeals against the
orders of NCLT. NCLAT acts as an intermediate appellate forum where appeals are heard after the
tribunal's decision. So, statement 3 is not correct.
○ NCLAT handles appeals against orders passed by the Insolvency and Bankruptcy Board of India
(IBBI), National Financial Reporting Authority and the Competition Commission of India (CCI). Its
decisions can be appealed to the Supreme Court.
So, only one of the above statements is correct.
Therefore, option (a) is the correct answer.
Relevance: The government is considering a proposal to increase the sanctioned strength of the National
Company Law Tribunal (NCLT) to 100.

Q82.
Answer: c
Explanation:
● India is a net exporter of wheat, rice and other food grains such as rye, maize, sorghum, buckwheat,
jawar, bajra, ragi and their import is negligible. In so far as pulses are concerned, the domestic demand
exceeds indigenous production, necessitating imports. So, points 1, 2 and 4 are correct.
● India is the world’s largest producer, consumer, and importer of pulses (peas, lentils, and beans), with
annual imports reaching $2.1 billion in 2021. So, point 3 is not correct.
Therefore, option (c) is the correct answer.
Relevance: The Union Minister of Agriculture & Farmers’ Welfare provided the details of import of food-grains
during the last three years in a written reply in Rajya Sabha recently.

Q83.
Answer: d
Explanation:
● A cheque is usually a written commitment made by the payer to the payee against a sum of money. The
payee, also known as the drawee, deposits this cheque in the bank. In an ideal situation, the payer’s
bank transfers the funds from the payer’s account to the payee.

47
Vajiram & Ravi Prelims Test Series (2024)
Full Length Test – 05 – (R5534)
● However, there are times when the payer’s bank or the payee’s bank refuses to honour this
commitment. The reasons for this ‘decline’ may vary. In such a case, the cheque bounces and is called a
‘dishonoured cheque’.
● A cheque can be dishonoured for the following reasons:
○ It could be because the issuer of the cheque did not have sufficient balance in the account. So,
point 1 is correct.
○ The signature on the cheque did not match exactly. So, point 2 is correct.
○ If the account number of the drawer of the cheque did not match. So, point 3 is correct.
○ Disfigured and damaged cheques may also be dishonoured by the bank. So, point 4 is correct.
○ If there is a problem with the date of issuing it.
○ If it has expired.
○ A cheque may bounce or sometimes, if the issuer may choose to stop the payment; in that case,
the cheque is considered dishonoured.
Therefore, option (d) is the correct answer.
Relevance: Use Net Banking or Mobile Banking to transfer funds to third-party accounts has reduced cheque
dishonour charges in recent times.

Q84.
Answer: c
Explanation:
● The Fifth Schedule of the Indian Constitution deals with the management and administration of
Scheduled Areas and Scheduled tribes where tribal groups predominate. Article 244(1) of the
Constitution of India states that the provisions of the Fifth Schedule shall apply to the administration and
control of the Scheduled Areas and Scheduled Tribes in any State other than the States of Assam,
Meghalaya, Tripura, and Mizoram. The President is empowered to declare an area to be a Scheduled
Area.
● The Dhebar Commission (1960-61), laid down the following criteria for declaring any area as a
‘Scheduled Area’ under the Fifth Schedule:
○ Preponderance of tribal population, which should not be less than 50 percent. So, point 1 is
correct.
○ Compactness and reasonable size of the area.
○ Underdeveloped nature of the area. So, point 2 is correct.
○ Marked disparity in the economic standard of the people as compared to the neighbouring Areas.
So, point 3 is correct.
● Later, a viable administrative entity, such as a district, block, or taluk, has been also identified as an
important additional criterion. According to the Union Ministry of Tribal Affairs, these criteria are not
spelt out in the Constitution of India but have become well established.
So, all of the above criteria are considered for declaring an area as a ‘Scheduled Area’ in India.
Therefore, option (c) is the correct answer.

48
Vajiram & Ravi Prelims Test Series (2024)
Full Length Test – 05 – (R5534)
Q85.
Answer: d
Explanation:
● The nature of the bond between the superior and inferior vassals and between the vassal and lord is
rather uncertain in the Post Gupta period. Some insights into the lord-vassal relationship are provided
by the usage of panchamahashabda, which seems to have developed as a samanta institution in the
post-Gupta centuries. Many inscriptions show that a measure of the high feudal rank enjoyed by some
vassals was their investiture with the panchamahashabda by their overlords.
Therefore, option (d) is the correct answer.

Q86.
Answer: d
Explanation:
● Dayabhaga and Mitakshara schools of Hindu law focus on property inheritance and individual rights to
property.
● The school of Mitakshara advocates that the son shall have equal rights and ownership over the
father’s ancestral property. The co-parcener comes into existence on the birth of a son. Under the
Dayabhaga law, the son does not acquire any interest by birth in the ancestral property, but the son’s
right arises for the first time on the death of the father and property devolves by succession and not by
survivorship. So statement 1 is not correct.
○ According to Mitakshara law, the powers of the father are limited and qualified over joint family
property. Under Dayabhaga, the father has absolute power to dispose of all kinds of property,
separate as well as ancestral, by sale, mortgage or will. The son has no right to demand partition
during father’s lifetime.
● Rights of Women: In the Mitakshara system, the wife cannot demand partition. She, however, has the
right to a share in any partition affected between her husband and her sons. Under the Dayabhaga this
right does not exist for the women because the sons cannot demand partition as the father is the
absolute owner. So, statement 2 is not correct.
● In both the systems, in any partition among the sons, the mother is entitled to a share equal to that of a
son. Similarly, when a son dies before partition leaving the mother as his heir, the mother is entitled to
a share of her deceased son as well as a share in her own right when there is a partition between the
remaining sons.
Therefore, option (d) is the correct answer.
Knowledge Box

Mitakshara school of Hindu law:


● It is commonly followed in the western and southern regions of India. It originates from
a commentary written by Vijnaneswara in the 12th century on the Yajnavalkya Smriti.
● Coparcenary is a key concept. This means that ancestral property is owned jointly by all
male family members, who share equal rights to it.
● It also acknowledges daughters’ right to inherit property, though with some limitations.

49
Vajiram & Ravi Prelims Test Series (2024)
Full Length Test – 05 – (R5534)
Dayabhaga school of Hindu law:
● It mainly practised in the eastern parts of India, particularly in Bengal. It comes from a
commentary written by Jimutavahana in the 13th century on the Yajnavalkya Smriti.
● Unlike Mitakshara, Dayabhaga doesn’t consider coparceners. Instead, it emphasises
individual ownership of property. Inheritance of property in this school is based on the
principle of lineal succession, where the nearest male relative in the family inherits the
property.
● Daughters also have the right to inherit property under this school, but their rights are
more restricted and subject to specific conditions.

Q87.
Answer: c
Explanation:
● A major ruling group of the post-Mauryan period was the Kushanas. The Kushanas were a branch of the
Yueh-chi, a nomadic group of people who inhabited territories near Dunhuang. It spread to encompass
much of modern day Uzbekistan, Afghanistan, Pakistan and Northern India.
● During the Kushan rule, there was simultaneous rule of two rulers - one senior and a junior ruler which
was a form of hereditary dual rule. It appears that there was less centralization under these rulers. They
strengthened the satrap system of government adopted from the Sakas. So, statement 1 is correct.
● Apart from using the coins as a mode of projecting their claims for divinity, the Kushana rulers also
constructed dynastic sanctuaries Bagolango or Bogopouro (in Bactrian) or devakula (in Sanskrit/Prakrit
inscriptions) and they also became objects of divine worship. So, statement 2 is correct.
○ Vima Kadphises (a Kushan emperor from approximately 113 to 127 CE) began the construction
of two such centres of royal cult, one at Mat, near Mathura, the other in Surkh Kotal. Kanishka I
(reign 127–150 CE) continued and finished building the dynastic sanctuary at Surkh Kotal.
Therefore, option (c) is the correct answer.

Q88.
Answer: c
Explanation:
● Lord Mahavira’s immediate disciples were known as Ganadharas. These disciples were mendicants as
the Jina is and have followed him since before his enlightenment. So, statement 1 is correct.
● All Ganadharas possesed perfect knowledge called keval-gyan. They orally compiled the direct
preachings of Lord Mahavira into 12 main texts called sutras. These texts are called Angas (limbs or
main texts). These are the oldest religious scriptures and the backbone of Jain literature. So, statements
2 and 3 are correct.
So, all three of the above statements are correct.
Therefore, option (c) is the correct answer.

50
Vajiram & Ravi Prelims Test Series (2024)
Full Length Test – 05 – (R5534)
Q89.
Answer: d
Explanation:
● Dedicated to Sun God, Suryan, Suryanar Kovil is one among the nine revered Navagraha Temples in
Kumbakonam, Tamil Nadu. As per the inscriptions found in the temple, Suryanar Kovil was believed to
have been built during the reign of Kulothunga Choladeva (1060-1118 CE) in the 11th century. It was
known as Kulottungachola-Marttandalaya in those times. So, statement 1 is not correct.
○ The King has had a good relationship with the Gahadwal dynasty of Kanauj whose rulers were
ardent devotees of Sun God. Hence, Suryanar Kovil is deemed to be a manifestation of their
influence in South India. Later, the temple was extensively renovated by the Vijayanagara Empire.
● During the reign of Rajaraja Chola I (985 CE to 1014 CE), the texts of the Tamil poets Appar, Sambandar
and Sundarar were collected and edited into one compilation called Thirumurai. So, statement 2 is not
correct.
Therefore, option (d) is the correct answer.

Q90.
Answer: c
Explanation:
● Zakat was a religious tax imposed on wealthy Muslims. It was a financial term in Islam that requires
Muslims to donate a portion of their wealth to charity. It is one of the five pillars of Islam and is
considered a mandatory act for Muslims who have a minimum amount of wealth. It comprised tax on
flocks, herds, gold, silver, commercial capital, agricultural produce, etc. So, pair 1 is correctly matched.
● Khams represented one fifth of the booty acquired in war or mine or treasure trove (found) to be
handed over to the state. So, pair 2 is correctly matched.
● Kharaj was the tax on land. Initially this tax was not levied on muslims however due to the need of the
state for revenue it was later not practical to give immunity to muslims from the payment of this tax.
Theoretically, the holders of Kharaj land were to pay land tax whether land was cultivated by them or
not. So, pair 3 is correctly matched.
So, all three of the above pairs are correctly matched.
Therefore, option (c) is the correct answer.

Q91.
Answer: d
Explanation:
● The Sangama dynasty was the first dynasty to rule the Vijayanagara Empire, which lasted until around
1485. It was founded in the 14th century by two brothers: Harihara I and Bukka Raya I.
Therefore, option (d) is the correct answer.
Knowledge Box

List of rulers of Sangama Dynasty:


● Harihara I (1336–1356 CE)
● Bukka Raya I (1356–1377 CE)

51
Vajiram & Ravi Prelims Test Series (2024)
Full Length Test – 05 – (R5534)
● Harihara II (1377–1404 CE)
● Virupaksha Raya (1404–1405 CE)
● Bukka Raya II (1405–1406 CE)
● Deva Raya (1406–1422 CE)
● Ramachandra Raya (1422 CE)
● Vira Vijaya Bukka Raya (1422–1424)
● Deva Raya II (1424–1446 CE)
● Mallikarjuna Raya (1446–1465 CE)
● Virupaksha Raya II (1465–1485 CE)
● Praudha Raya (1485 CE), last ruler

Q92.
Answer: b
Explanation:
● Optical fibres are made of thin cylindrical strands of glass. The diameter of a typical fibre is close to the
diameter of a human hair. These fibres can carry information, such as text, images, videos, telephone
calls and anything that can be encoded as digital information. So, statement 1 is correct.
● When a beam of light falls on a glass surface, it passes through partially while the rest is reflected away.
When it passes through, its path bends because the refractive index of glass is different from that of air.
When a beam travels in the reverse direction, that is from glass to air, it’s possible that it won’t enter
the air. Instead, it will be completely reflected back within the glass. This phenomenon, known as total
internal reflection, is the basis of guiding light across long distances without a significant loss of optical
power. So, statement 2 is correct.
● In India, the Fibre Optics Laboratory at the Central Glass and Ceramic Research Institute, Kolkata, has
a facility to manufacture high-quality silica-based optical fibres. Today’s optical fibres have a typical loss
of less than 0.2 dB/km. So, statement 3 is not correct.
So, only two of the above statements are correct.
Therefore, option (b) is the correct answer.

Q93.
Answer: b
Explanation:
● Otolith rings, also known as "earstones," are hard, calcium carbonate structures primarily found in bony
fish (teleost fish) and some cartilaginous fish (elasmobranchs), such as sharks and rays. They are not
present in all vertebrates. So, statement 1 is not correct.
○ These rings are similar to tree rings and can be used to determine the age of a fish. The rings can
also reveal information about the fish's environment, such as the temperature it experienced and
how quickly it converted food into energy.

52
Vajiram & Ravi Prelims Test Series (2024)
Full Length Test – 05 – (R5534)
● Otolith rings in fish are used to understand the impacts of climate change on aquatic environments by
providing information on a fish's age, growth rates, and historical water temperature variations. This
helps scientists track changes in fish populations and ecosystems over time, aiding in the assessment of
climate-related impacts. So, statement 2 is correct.
Therefore, option (b) is the correct answer.
Relevance: Scientists have used otoliths to study how lake trout respond to rising temperatures in Alaska.

Q94.
Answer: a
Explanation:
● The Global Positioning System (GPS) is a network of satellites and receiving devices that can determine
the location of something on Earth.
● GPS satellites are placed in Medium Earth Orbit (MEO). They orbit the Earth at an altitude of about
20,200 kilometers and circle the Earth every 12 hours. The MEO provides a good balance between
coverage and signal strength. So, statement 1 is not correct.
● A GPS device needs to read signals from at least four satellites to calculate a location. The signals from
four satellites allow for precise triangulation and calculation of a position in three dimensions. So,
statement 2 is not correct.
● GPS receivers use radio waves to determine the location, speed and direction of objects. Because radio
waves travel at a constant speed, the receiver can use the time measurements to calculate its distance
from each satellite. So, statement 3 is correct.
Therefore, option (a) is the answer.
Knowledge Box

● The GPS consists of three segments: the space segment, the control segment,
and the user segment. The U.S. Space Force develops, maintains and operates
the space and control segments.
○ Space Segment: It consists of a nominal constellation of 24 operating
satellites that transmit one-way signals that give the current GPS satellite
position and time.
○ Control Segment: It consists of worldwide monitor and control stations
that maintain the satellites in their proper orbits through occasional
command maneuvers and adjust the satellite clocks. It tracks the GPS
satellites, uploads updated navigational data, and maintains health and
status of the satellite constellation.
○ User Segment: It consists of the GPS receiver equipment, which receives
the signals from the GPS satellites and uses the transmitted information
to calculate the user's three-dimensional position and time.

53
Vajiram & Ravi Prelims Test Series (2024)
Full Length Test – 05 – (R5534)
Q95.
Answer: b
Explanation:
● Niemann-Pick disease refers to a group of rare, inherited metabolic disorders in which abnormal
amounts of lipids (fatty materials such as waxes, oils, and cholesterol) build up in the brain, spleen, liver,
lungs, and bone marrow.
● It is an Autosomal Recessive (AR) disease, which means both the parents must be affected or carriers
of the genes for the disease to pass on to their children. Carriers of an AR disease do not show signs and
symptoms of the disease, but they can pass the defective gene to the next generation. So, statement 2
is correct.
● Niemann-Pick disease is currently not recognized as a rare disease under the National Policy for Rare
Diseases, 2021. So, statement 1 is not correct.
○ Once a disease is recognized as a rare disease under the National Policy for Rare Diseases, 2021,
financial support of up to 50 lakh will be provided for the treatment.
Therefore, option (b) is the correct answer.
Relevance: Parents of children with Niemann-Pick disease, a rare genetic disorder, are urging the Government
of India to include the ailment in the National Policy for Rare Diseases.

Q96.
Answer: b
Explanation:
● Varunastra is a heavy weight, electrically propelled anti-submarine torpedo capable of targeting quiet
submarines, both in deep and shallow waters. It was developed by the Naval Science and Technological
Laboratory (NSTL) of the Defence Research and Development Organisation (DRDO). So, statement 3 is
correct.
○ It is capable of being launched from surface ships and submarines. So, statement 1 is not
correct.
● It is the only torpedo in the world to have a GPS (Global Positioning System)-based locating aid. It has
conformal array transducers that allow it to look at wider angles than most common torpedoes. So,
statement 2 is correct.
Therefore, option (b) is the correct answer.
Relevance: The Indian Navy successfully test-fired the Varunastra, an indigenously designed and developed
heavy-weight torpedo (HWT).

Q97.
Answer: d
Explanation:
● Medicinal and aromatic plants are botanical raw materials, also known as herbal drugs, that are
primarily used for therapeutic, aromatic, and/or culinary purposes as components of cosmetics,
medicinal products, health foods and other natural health products.
● In its quest to promote the diversification of agriculture, the Government of India is promoting the
cultivation of aromatic plants. With the help of the Central Institute of Medicinal and Aromatic Plants
(CIMAP) in Lucknow. The Government has recently introduced different aromatic plants such as
54
Vajiram & Ravi Prelims Test Series (2024)
Full Length Test – 05 – (R5534)
menthol mint (the CIM-Unnati variety), rosemary (Hariyali variety), patchouli (CIM-Samarth), damask
roses (Ranisahiba), chamomile and geraniums (CIM-Bharat) in the Nabarangpur district of Odisha. So,
points 1, 2, 3 and 4 are correct.
So, all four of the above are aromatic plants.
Therefore, option (d) is the correct answer.

Q98.
Answer: a
Explanation:
Alliance for Transformative Action on Climate and Health (ATACH):
● It is a World Health Organization (WHO) initiative that aims to build sustainable and climate-resilient
health systems. ATACH was launched at the 2021 United Nations Climate Change Conference or the
Conference Of Parties 26 (COP26). It works to realize the ambition set at COP26 to build climate resilient
and sustainable health systems.
● It is an informal voluntary network for Participants to exchange views, share information and enhance
technical and political cooperation. It is not a distinct legal entity and it derives its legal status from
WHO. Thus, it is administered by WHO, which provides its Secretariat.
Therefore, option (a) is the correct answer.
Relevance: WHO has launched the Alliance for Transformative Action on Climate and Health (ATACH) to build
climate resilient and sustainable health systems.

Q99.
Answer: d
Explanation:
● Cloud seeding is a weather-modification method. Its purpose is to induce rain and snowfall which may
lead to reduced droughts. It works on the principle of nucleation. The intent of seeding clouds is to
introduce aerosol or nucleating particles into a cloud. That introduction may influence the natural
development of cloud particles.
● Cloud Seeding Agents (CSA): Chemicals that are used to induce precipitation are known as CSA.
Following agents are used as CSA:
○ The most common of these substances is silver iodide. So, point 1 is correct.
○ Liquified propane. So, point 2 is correct.
○ Dry ice (frozen carbon dioxide). So, point 3 is correct.
○ Potassium iodide. So, point 4 is correct.
○ Sodium chloride
○ Calcium chloride
So, all four of the above can be used as Cloud Seeding Agents (CSA).
Therefore, option (d) is the correct answer.

55
Vajiram & Ravi Prelims Test Series (2024)
Full Length Test – 05 – (R5534)
Q100.
Answer: d
Explanation:
● The Black-spotted croaker, also known as Ghol locally, has been recently declared as the ‘State fish’ of
Gujarat. It is termed as ‘lottery’ and ‘Sea Gold’ for fishermen, both for its body parts’ culinary delights
and their medicinal value.
● Commercial importance of Black-spotted croaker:
○ Medicinal importance: Its air bladder which is cut open from the stomach and dried is mainly
exported to China, Hong Kong and other Asian countries where it is in high demand for its
medicinal values.
■ It is believed to be rich in magnesium, iodine, Omega-3 and iron, among other nutrients,
which are believed to help in maintaining eyesight, tone muscles, build IQ for infants and
prevent aging. Its air bladder is a particularly prized possession for medicinal value.
○ Economic importance: Its meat is exported as frozen fillet or whole fish to European and Middle-
Eastern countries.
● Distribution of black-spotted croaker:
○ Indo-West Pacific: west coast of the Persian Gulf and along the coasts of India and Sri Lanka,
north to Japan and south through the Philippines and Borneo to New Guinea and northern
Australia.
Therefore, option (d) is the correct answer.
Relevance: Gujarat recently declared the Black-spotted croaker(Protonibea Diacanthus), locally known as the
Ghol fish, as the state fish.

56
Vajiram & Ravi Prelims Test Series (2024)
Full Length Test – 05 – (R5534)

You might also like